Вы находитесь на странице: 1из 54

ARAT:

Directions: Each question contains six statements followed by four sets of combinations of three. Choose the set
in which the statements are logically related
1. A. All snakes are reptiles.
B. All reptiles are not snakes.
C. All reptiles are cold blooded.
D. All snakes lay eggs.
E. All reptiles lay eggs.
F. All snakes are cold blooded.
* ADE * BED
ABE

* ACF

2. A. All maps are caps.


B. All caps are traps.
C. No caps are maps.
D. No caps are traps.
E. All maps are traps.
F. All traps are maps.
* ABE * FAB
* AFE

* DBE

3. A. P is a hero.
B. Some heroes have fans.
C. Some villains have fans.
D. P may have fans.
E. P is not a villain
F. Some villains have no fan clubs.
* FCA * ACF
* CFB

* BAD

4. A. All books are biographies.


B. Some books are popular.
C. Britannica is a biography.
D. Britannica is a popular book.
E. Some popular books are biographies.
F. Britannica is not popular.
* ACD * ABE
* DCA
* EDC
5. A. X is dumb.
B. dumb people cant speak.
C. Y is deaf.
D. Y cant say I spoke.
E. Y cant say I heard.
F. X cant say I spoke.
* DCA * CDF * ABC * BAF
6 . A. Army men wear uniforms.
B. Soldiers do not wear uniforms.
C. Amar Singh is a Soldier.
D. Amar Singh may wear uniform.
E. Some Soldiers are army men
BCD
ECD
ABC

None of these

8. A. All Inhabitants of Andaman are tribals


B. Mogambo is a tribal
C. Mogambo is an inhabitant of Andaman
D. Most Negroes are Africans.
E. Some inhabitants of Andaman are Africans.
ABC
BCA
AED

None of these

13/15. A. the Japanese are techno-savvy.


B. Suzuki is techno-savvy
C. Same techno-savvy are illiterate.
D. Suzuki is Japanese.
E. Suzuki may be illiterate.
ABD BAD
BCD
DCB

14/15. A. Many Bangaloreans are friendly


B. Some friendly persons may be accommodative.
C. All accommodative persons are tolerant
D. Some tolerant persons are friendly.
E. All Bangaloreans are tolerant.
CDB
DEB
ADE
BCA
15/15 A. Parasites live on other animals.
B. Most parasites are invisible
C. Most animals are visible
D. E coli is parasite.
E. E coli may not be visible
DEB ADE
BDE
None of these

Direction: Questions are based on the graph given below


The graph indicates the target and achieved production level along with the number of defects produced for a
company
1/15. What is the change in percentage defectives (of production achieved) between 1987 to 1990
6% increase

4% increase

6% decrease

1.5% decrease

Each of the questions below is followed by two statements by two statements. Mark,
1. If statement A alone answers the question.
2. If statement B alone answers the question.
3. If both statements A and B are required to answer the question
4. If both statements A and B are insufficient to answer the question
6/15. There are three numbers a,b and c. Are these numbers in arithmetic progression?
A. a+c=2b
B. a+b<c
7/15. If m and n are integers, what is the value of m+n?
A. (x+m) (x+n)=x2+5x+mn and x=/0
B. mn=4
8/15. if x5=x3 what is the value of x
A. x>-3
B. x<0
9/15. The figure shows the floor plan and inside dimensions of the ground level of a building if all the lines that
intersect meet at right angles, what is the floor area of this of level of the building?
A.QR-4mts
B.PQ-3mts

10/15. The 9 squares in the figures are to be filled with Xs and Os with only one symbol in each square. How
many squares will contain an X?
A.
More than 1/2 the number of squares will contain an O
B.
Each of the 4 corner squares will contain an X.

Directions: Each question is followed by 2 statements. I and II. Mark the answer

A.
B.
C.
D.

If the question can be answered by using either one of the statements alone, but cannot be
answered using the other statement alone.
If the question can be answered by using either statement alone.
If the question can be answered using both statements together, but cannot be answered using
either statements alone.
If the question cannot be answered even by using both statements together.

1/15. Is x positive? ([x] Refers to the greater integer function less than x).
I.
[x] * x = 0
II.
[x] - |x| <= 2|x|
2/15 is x>1?
I.
II.

1/ x < 1
X2< 1

3/15. What is the value of x (x Real Numbers)?


I.
X3 = 1
II.
|X| = 1
4/15. Is X3>0 ?
I.
X =Y + 5
II.
Y2 = 4
5/15. Is x greater than 3.5?
I.
2x > 6
II.
3x < 10.9

Direction: Questions are based on the following data

Credit sales for the last quarter of the previous year were worth Rs. 7.5 lakhs.
6/15. The film does not charge the customers any interest on credit sales. The prevailing interest rate is 1.5% per
month. Which quarter has resulted in the highest loss due to credit sales. ?
1.

Qtr 4

2. Qtr 3

3. Qtr 2

4. Qtr 1

7/15. If 1.75% of all credit sales add up to as bad debts(un recoverable). How much has been the overall loss due
to bad debts for the year. ?
1.
Rs 32,250
2. Rs. 28,218.75
3. Rs 31,250 4. Rs 32,625
8/15. Collection of credit sales of a quarter is done in the next quarter. Which quarter had the highest cash
inflows?
1.
Qtr 4 2. Qtr 2
3. Qtr 2
4. Qtr 1
9/15. Average sale/month (ignoring bad debts) for the year was?
1.
Rs 2,08,333.33 2. Rs. 6,55,000
3. Rs. 3,13,500
4.Rs. 3,26,250
10/15. What has been the percentage change in the sales from the 1 st quarter to the 4th quarter?
1.
35% 2. 64%
3. 14%
4. 120%

Directions: Given below are the two statements followed by the two conclusions. Disregarding commonly
known facts, taking the two statements to be thoroughly true, mark your answer as.
A.
If only I follows.
B.
If only II follows.
C.
If both I and II follows.
D.
None follows.
11/15. Statements:
I.
SaniaMirza is a tennis player.
II.
Tennis players are tall.
Conclusions:
I.
II.

All tall players are tennis players.


Saniamirza is tall.

12/15. Statements:
I.
Some tables are stools.
II.
Some stools are chairs.
Conclusion:
I.
Some tables are stools.
II.
Some stools are tables.
13/15. Statements:
I.
All I.A.S have good general knowledge.
II.
Jatin doesn't have good G.K.
Conclusions:
I.
Jatin is not an I.A.S.
II.
Jatin should watch news daily.
14/15. Statements:
I.
All long boots are made of leather.
II.
II. No leather is wooden.
Conclusions:
I.
No long boot is wooden.
II.
II. No wooden thing is long boots.
15/15.Statements:
I.
All bats are blind.
II.
II. All snakes are blind.
Conclusions:
I. Some snakes are bats.
II. No snake is bat.
Directions: Each question contains six statements followed by four sets of combinatins of three. Choose the set
in which the statements are logically related.
1/15.
A. Some locks are keys
B. All keys are hooks
C. Some locks are hooks
D. Some hooks are locks
E. No lock is a hook
CDE
ECD
CDA
ABC
2/15.
A. All passengers are messengers
B. Some passengers are intruders
C. All intruders are passengers
D. Some messengers are passenger
E. No intruder is a passenger
ABC
DFE
ABD
3/15.
A. Some diamonds are bracelets

ADC

NONE

B. All rings are diamonds


C. All are bracelets
D. No diamond is a bracelet
E. A few bracelets are diamonds
F. No bracelet is a ring
BCA
DFA
CDE
EFD
NONE
4/15.
A. No young is prudent
B. No prudent is young
C. All students are young
D. No student is prudent
E. All young are students
F. Some young are not prudent
EFD
CAD
ABD
ECB
5/15.
A. All actors are stars
B. A few actors are planets
C. Some planets are actors
D. All stars are planets
E. No planet is a star
F. No star is an actor
CEF
AFC
ADC
BED
DIRECTIONS: Study the following table carefully to answer the questions.

6/15.
What is the ratio between the number of students studying computers and commerce respectively ?
A. 4:11
B. 9:11
C. 11:9
D. None of these
7/15.
Number of female students for which of the following disciplines is exactly the same ?
A. Engineering and Management
B. Engineering and science
C. Computers and Arts only
D. Engineering, Management and Economics
8/15.
What is the difference between the male and female students studying arts?
A. 118
B. 256
C. 128
D. None of these
9/15.

What is the total number of male studying commerce and economics?


A. 516
B. 640
C. 584
D. None of these
10/15.
How many male students are studying science?
A. 691
B. 720
C. 432
D. None of these
Directions: the problems below contain a question and two statements giving certain data. You need to decide
whether the data given in the statements are sufficient for answering the question. The correct answer is:
1. If statement(I) alone is sufficient
2. If statement (II) alone is sufficient
3. If both statements(I) and (II) are together sufficient but neither of the statements alone is sufficient
4. If each statement alone is sufficient
5. If statements (I) and (II) together are not sufficient
11/15.
Is n > p? n and p are real number?
I. m/n = n + p
II. n > m
12/15.
Are two triangles congruent?
I. Both triangles are isosceles
II. Both triangles have same perimeters
13/15.
How many boys and girls are there in the family of 7 children?
I. Each boy in the family has as many sisters and brothers
II. Each of the girls in the family has twice as many as sisters
14/15.
If rope is cut 3 pieces of unequal length, what is the length of the shortest of these pieces?
I. The combined length of the longer two pieces of rope is 10 meters
II. The combined length of the shorter two pieces of rope is 9 meters
15/15.
What is the digit in the units place of a number of two digits?
I. The sum of the digits of the number is 16
II. If the digits are interchanged the number remains the same
Directions: Use the information given below:
A manufacturer produces widgets and sends them to the market in lots of 1000 units. The manufacturing process
sometimes produces defective units. Assume `p` denotes the proportion of defectives in a lot. If
defective widget found before it is sent to the market, it could be corrected at a cost. On the other hand,
the manufacturer has to
pay a penalty if a defective units is received by customer. The manufacturer was considering which of the three
alternative methods of inspection procedures he should choose. The graph below shows the cost curve
for each of the alternatives as a function of proportion (p) in a lot.
INSPECTION COST CURVES

PROPORTION OF DEFECTIVES:

(6)1/15.
IF P = 0.05, the best alternative is
A. Alt 1 only
B. Alt 1 or Alt 2
C. Alt 3 only
D. Alt 1 or Alt 3
(7)2/15. Alt 2 is the best if
A. P < 0.05
B. 0.05 < p < 0.2
C. P > 0.2
D. None of these
(8)3/15. If p is <0.2 the best alternative is
A. Alt 1 only
B. Alt 2 only
C. Alt 3 only
D. Not alt 3
(9)4/15
Alt 3 is the best if
A. P < 0.05
B. 0.05< p <0.2
C. P > 0.2
D. None of these
(10)5/15 if p Is greater than 0.1 but less than 0.2, the best alternative is
A. Alt 1 only
B. Alt 2 only
C. Alt 1 or Alt 2
D. Alt 2 or Alt 3
(1)6.
Directions: each question contains six statements followed by four sets of combinations of three. Choose the set
in which the statement are logically related.
A. Some cabinets are made of wood
B. All wood things are not hard
C. Some chairs are cabinets
D. Some chairs are made of wood
E. All cabinets are hard
F. Some chairs are hard
A. ABC B. CDF C. CEF D. EFC
(2)7
Directions: each question contains six statements followed by four sets of combinations of three. Choose the set
in which the statement are logically related.
A. A thesis is considered exceptional for its originality
B. Some thesis are original
C. All thesis are not exceptional
D. Sameera`s thesis is exceptional
E. All thesis are not considered original
F. Sameera`s thesis is original
A. FBE B. ACD C. DBC D. ADF
(3)8/15
Directions: each question contains six statements followed by four sets of combinations of three. Choose the set
in which the statement are logically related.
A. Cricket is played by gentlemen
B. Some cricketers are not gentlemen
C. Avinash is a good cricketer
D. Some gentlemen area not cricketers
E. Avinash is a gentleman
F. Avinash is not a gentleman
A. ACF B. ACE C. BCF D. BCE
(4)9/15.
Directions: each question contains six statements followed by four sets of combinations of three. Choose the set
in which the statement are logically related.
A. No prisoner is a criminal

B. Pilot is not criminal


C. Pilot is a prisoner
D. Pawan is not a prisoner
E. Pawan is a criminal
F. Some criminals are prisoners
A. ABE B. ABC C. ADE D. ACB
(5)10/15.
Directions: each question contains six statements followed by four sets of combinations of three. Choose the set
in which the statement are logically related.
A. Some collectors are cops
B. Most cops are athletes
C. Some athlete men are divers
D. No diver is a boxer
E. Some boxers re not divers
F. Some athlete men re not boxers
A. DCE B. CDF C. BAF D. ABC
11/15
Directions: each of the following questions below consists of a question and two statements numbered I and II
given below it. You, have to decide whether the data provided in the statements are sufficient to answer
the question. Read both the statements and mark the answer 1. If the data in statement I alone are
sufficient to answer the question, while the data in statement II alone are not sufficient to answer the
question If the data in statement II alone are sufficient to answer the question, while the data in
statement I alone are not sufficient to answer the question If the data either in statement I alone or in
statement II alone are sufficient to answer the question If the data even both the statements I and II are
not sufficient to answer the question If the data in both the statements I and II together are necessary to
answer the question
What is the speed of the train whose length is 210 metres?
I
The train crosses another train of 300 metres length running in opposite direction in 10 seconds
II
The train crosses another train of 300 metres running in the same direction at the same speed in
30 seconds
12/15.
What is the volume of a cube?
I
The area of each face of the cube is 64 square metres
II
The length of one side of the cube is 8 metres
A. 1 or 2 B. 3 C. 4 D. 5
13/15.
What is the simple rate of interest?
I
The total interest earned was RS. 4000/II
The sum was invested for 4 years
A. 1 or 2 B. 3 C. 4 D. 5
14/15.
What is the number?
I
40% of the number is equal to 2/5th of the number
II
20% of the number is equal to 30% of 40
A. A B. B C. C D. D
15/15.
What is the two digit number?
I
The difference between the two digits is nine
II
The sum of the two digits is nine
A. 1 or 2 B. 3 C. 4 D. 5
Direction: Refer the following data and answer the questions.

Total number of students = 8000


Male : Female
Arts
2:3
Medicine
1:1
Management
9:7
Engineering
7:5
Science
Commerce

4:5
3:5

1/15. Total number of female students studying Engineering and Medicine is


A.
1280
B.
5000
C.
820
D.
480
2/15. How many male students are there in the arts stream?
A.
320
B.
480
C.
800
D.
720
3/15. What is the total number of male students studying Commerence?
A.
1280
B.
1440
C.
1650
D.
None
4/15. What is the difference between the number of students studying Arts and School?
A.
480
B.
640
C.
800
D.
320
5/15. Number of female students studying Management is what percentage of the total number of
students in the University?
A.
27
B.
12
C.
9
D.
None
Directions: Each of the following questions consists of two statements each followed by two conclusions I and
II. Take them as true even if the given two statements are not consistent with the known facts and
ascertain as to which of the two conclusions from among the given statements comes out to be logical.
1.
If only the conclusion I is true.
2.
If only the conclusion II is true.
3.
If either conclusion I or the conclusion II is true.
4.
When neither the conclusion I nor the II is true.
6/15. Statements:
All papers are files.
Some files are pens.
Conclusions:
I.
No paper is a pen.
II.
Some pens are papers.
7/15. Statements:

Some clips are small.


Everything is either small or big.
Conclusions:
I.
All bigs are small.
II.
All smalls are clips.
8/15. Statements:
All books are pencils.
All pencils are pins.
Conclusions:
I.
II.

Some pins are books.


No pin is a book.
9/15. Statements:
All books are pens.
All pens are pencils.

I.
II.

Conclusions:
All books are pencils.
All pencils are books.
10/15. Statements:
Some books are toys.
No toy is red.

I.
II.

Conclusions:
Some books are red.
Some books are not red.
Directions: Each questions is followed by two statements, I and II. Mark the answer

1.
2.
3.
4.

If the question can be answered by using one of the statements alone, but cannot be answered
using the other statement alone.
If the question can be answered by using either statement alone.
If the question can be answered by using both statements together, but cannot be answered using
either statement alone.
If the question cannot be answered even by using both statements together.

11/15. If x is an odd integer, is y an odd integer?


I.
The average of x and y is an odd integer.
II.
The average of x, y and (y+1) is an integer.
12/15. If x < y < z are odd integers, are they consecutive odd integers?
I.
z x = 4.
II.
Y is the average of x and z.
13/15. What is the value of the positive integer N?
I.
N + 3 is divisible by both 6 and 9.
II.
N < 20.
14/15. What is the digit in the units place of a number with 2 digits?
I.
The sum of the digits of the number is 16.
II.
If the digits are interchanged, the number remains the same.
15/15. What is the value of the two digit number m?
I.
The difference between the 2 digits is 3.
II.
The sum of the 2 digits is 5.

-------------------------------------------------------------------------------------------------------------------------------------------------------------MCTLR:
There are three small tables in an ice-cream parlour Nirulas. Table 1 and 2 can seat three people each while,
Table 3 can seat only 1 person. Seven children- A, B, C, D, E, F and G are to be seated around these
tables. Who sits with whom and at which table are determined by the following conditions.
- C does not sit as the same table as G.
- E does not sit as the same table as D.
- F does not sit as the same table as C.
- A does not sit as the same table as B.
- G does not sit as the same table as F.
1. Which of the following is not the possible situation?
* A sits on table 1
* B and F sits on table2
* C sits on table 2
* D sits on table 3
* A and E sits on table 1
2. If C shares a table with E and G shares a table with A, then which of the following could be true?
* E shares table1 with B
* G shares table2 with E
* F shares table1 with B
* B shares table3 with A
* D shares table2 with C
3. Which list of the following are those children who can never share a table, and also it is not possible for any
two of them to share a table?
* C,E,F
* D,E,F
* C,G,F
* G,F,B
* C,D,A
4. If C sits with D and A cannot sit with F, also children who have vowel names has to be seated together, then
who has to sit alone?
*G
*F
*C
*B
*D
5. Which of the following is a list of children who could sit together at table2?
* B,F,C
* G,E,A
* C,G,D
* F,D,E

* B,E,A

6. Aman was asked to draw all the possible diagonals of an octane. The number of diagonals by Aman is:
* 18
* 22
* 24
* 26
* None of these
7. A bag contains 10 toys out of which 3 are rectangular and rest are spherical. Ankit was asked to draw a
random sample of 6 toys from the bag. The sample can have atmost 2 rectangular toys and it must not
contain all the toys of same shape. How many such samples can be drawn by Ankit?
* 172
* 168
* 166
* 188
* None of these
8. Two friends Aman and Ankit run around a circular track of length 430 meters, starting from the same point,
simultaneously and in the same direction,. Aman, who runs faster, meets Ankit for the first time in the
middle of the 5th round. If Aman and Ankit were to run a 3 kn race long race, how much start in metre,
in terms of distance, should Aman give Ankit so that they finish the race in exactly the same time?
* 555.56m
* 666.67m
* 777.78m
* 888.89m
* None of
these
9. If in certain language, sing means tkqt and dance means ecqgi, then what does play mean?
* Pmce
* Qndc
* Bqdk
* Nqmc
* Qnde
10. What is next alphabet in the sequence
A,C,F,J,O,.......
*T
*S
*U

*V

*X

1/10. How many different can be formed of the same size using letters from the word INFOSYS, such
that the two S cannot be adjacent to each other?
2520

750

2/10 Find the next item


4,25,121,289,529..
841
900

5040

1800

1600

961

784

1024

3/10 Nakul can do a piece of work in 9 days and Naudy can do the same piece work in 10 days. In how
many days will they complete 40 parts of the work out of 90 parts if they work alternatively?
Assume the Nakul will work on the first day.
21/5 days
5 days
4 days
6 days
None of these
4/10. Aman had 6 bags labelled A, B, C, D, E and F. Each bag is to contain either a red or green ball in
such a way that at least 1 bag contains a green ball and the bags containing red balls are
consecutively labelled. The total number of ways in which this can be done is:
21
25
23
31
None of these
5/10. If EAT+EAT+EAT= BEET, where every letter represents a unique digit (0-9), what is TEE +TEE?
(Value of T is 0)
BAA
TAA
TA
BET
TAB
Seven friends are standing in a row Akta, Binal, Chanda, Diya, Esha, Fenny and Gargi. The following
conditions apply:-Diya stands before Binal.
-Chanda stands after Esha.
-Fenny stands after Binal but before Chanda.
-Gargi stands before Akta but after Diya.
-Aktastandsa after Fenny.
6/10. If Esha stands second, each of the following must be true EXCEPT:
Binal stands third or fourth
Fenny stands fourth or fifth
Diya stands first.
Chanda stands first.
Akta stands sixth or seventh.
7/10 which of the following MUST be False?
Akta stands fifth.
Fenny stands fourth.
Chanda stands last.
Diya stands third
Gargi stands sixth
8/10 if Binal stands before Esha, which of the following Must be true?
Esha stands third.
Fenny stands third.
Chands stands last.
Fenny stands fourth.
Diya stands first.
9/10. Which of the following could be true?
Fenny stands before Diya
Chanda stands before Diya
Akta stands before Diya
Chanda stands before Gargi
Chanda stands before Binal
10/10. Which of the following could be the order in which they are standing?
Diya,Binal,Fenny,Gargi,Akta,Chanda,Esha
Binal,Diya,Gargi,Esha,Fenny,Akta,Chanda
Esha,Diya,Binal,Chanda,Fenny ,Gargi, Akta
Esha, Diya,Binal,Chanda,Fenny,Gargi,Akta
Esha,Diya,Binal,Gargi,Akta,Fenny,Chanda

Permutations and combinations:


1/10: 3 out if 10 different hats are to be chosen for the various characters in a school drama. In how many ways
this can be done?
10
1.
C3
10
2.
P3
3.
242.
4.
340.
5.
None of these.
2/10. If in a certain language, clear water means dnhewxcwio and game pass means hcplqcvw means
hcpiqcvw, then what does have fun mean?
1.
Icyihvo
2.
Icxigrl
3.
Icyiefg
4.
Icyigwq
5.
Iczqmwj
3/10. An arrangement was to be made for a batch photograph. The 7 girls will be sitting in the front row while
18 boys will be standing behind the girls. The middle two position in the second row is reserved for the
two shortest boys. In how many ways the arrangement can be done.?
1.
16! * 7!
2.
16! * 5! * 84
3.
(18! * 7!) / 2!
4.
16! * 5!
5.
None of the above
4/10. Two friends Piyush and Dipak run around a circular track of length 430 metres, starting from the same
point, simultaneously and in the same direction. Piyush, who runs faster.meetsDipak for the first time in
the middle of the 4th round. If Piyush and Dipak were to run a 5 km long race, how much start, in terms
of distance, should Piyush give Dipak so that they finish the race in the exactly same time?
1.
666.67 m
2.
1428.57 m
3.
714.285 m
4.
718.256 m
5.
None of these
5/10. What is the next number in the sequence?
1,3,7,15,,31,.
1.
61
2.
65
3.
64
4.
63
5.
60
The testing team of a project meet up for a review. Ethan is responsible for making the members sit for the
meeting in a row.The team consists of a moderator, an author, a scribe, an expert and a board
member.The team members are named Gilly, Henry, Jack, Kane and Larry.
The moderator must sit in the middle, in seat number 3
Jack sits on either end of the row
Larry is the author or a scribe
The scribe and the expert sit on either side of the moderator.
Henry. who is not the moderator, sits between Kane and Jack.
The moderator does not sit next to Larry.
Gilly .the expert. sits in seat number 4.
6/10. Which of the following is a possible seat for Henry?
1.
Seat 1 2. Seat 2
3.Seat 3
4. Seat 4
7/10. Who is the moderator?
1.
Larry 2. Gilly
3. Henry
4. Kane

5. Seat 5
5. Jack

8/10. Who is seated on seat no 2?


1.
Author 2. Jack
3. Board Member
4. Scribe
5.Kane
9/10. If Board Member has to be seated next to Scribe, then who will be seated on either side of expert?
1. Larry and Henry
2. Jack and Larry
3.Kane and Larry
4. Gilly and Larry
5. Kane and Henry.

10/10. What role does Jack play?


1.
Moderator
Member

2. Author

3.Scribe

4.Expert

5.Board

1/10.
If the 6 digit numbers formed from the digits 1,2,3,4,5,6 are arranged in the increasing order (repetition is not
allowed), then 291th term will be
A. 34126 B. 341265 C. 321456 D. 341526
2/10.
A 20 litre mixture of milk and water contains milk and water in the ratio 3:2. 10 litres of the mixture is removed
and replaced with pure mils and operation is repeated once more. At the end of the two removals and
replacement, what is the ratio of the milk and water in the resultant mixture? A. 9:1 B. 17:3 C. 3:17 D.
5:3
3/10.
Find the next term in the series: 15, 29, 56, 108, 208, .? A. 112 B. 386 C. 400 D. 416
4/10.
What will be the next term of the following sequence? A1E, D2C, G2A, J4Y, M8W, ? A. R16U B. P16T
C. R32T D. P32U E. None of these
5/10.
How many 10 digit numbers can be formed using digits 1 and 9? A. 10! B. 10^2 C. 2^10 D. None of these
6/10.
Exactly three doctors- Cameron, Foreman, and chase have been allotted shifts for their visit in the paediatric
ward on Monday, Tuesday and Wednesday -Each doctor visits at least once in the three days but never
more than once on a given day -Every day at least one doctor pays a visit to the ward and no two visit
simultaneously -On Monday foreman is on duty, and nobody has a shift after him on that day. -On
Tuesday either Cameron or chase, but not both, is on duty and there is nobody`s shift after that -On
Wednesday either Cameron or foreman, but not both, is on duty and there is nobody`s shift after that
6. Which one of the following could be complete and accurate order of the shifts?
A. Monday: chase, foreman; Tuesday : foreman ; Wednesday : foreman
B. Monday : foreman ; Tuesday: Cameron, chase; Wednesday : chase, the Cameron
C. Monday : foreman; Tuesday : chase; Wednesday: chase, Cameron
D. Monday : Cameron, foreman, chase; Tuesday : chase; Wednesday : Cameron
E. Monday: Cameron, foreman; Tuesday : chase, foreman; Wednesday : foreman
7/10.
Exactly three doctors- Cameron, Foreman, and chase have been allotted shifts for their visit in the paediatric
ward on Monday, Tuesday and Wednesday -Each doctor visits at least once in the three days but never
more than once on a given day -Every day at least one doctor pays a visit to the ward and no two visit
simultaneously -On Monday foreman is on duty, and nobody has a shift after him on that day. -On
Tuesday either Cameron or chase, but not both, is on duty and there is nobody`s shift after that -On
Wednesday either Cameron or foreman, but not both, is on duty and there is nobody`s shift after that
7. Which one of the following CANNOT be true?
A. Foreman is the last doctor on duty each day
B. Chase is on duty on all three days
C. Cameron is the second person to be on duty each day
D. A different doctor has the first shift each day E. A different doctor has the last shift each day
8/10.
Exactly three doctors- Cameron, Foreman, and chase have been allotted shifts for their visit in the paediatric
ward on Monday, Tuesday and Wednesday -Each doctor visits at least once in the three days but never
more than once on a given day -Every day at least one doctor pays a visit to the ward and no two visit
simultaneously -On Monday foreman is on duty, and nobody has a shift after him on that day. -On
Tuesday either Cameron or chase, but not both, is on duty and there is nobody`s shift after that -On
Wednesday either Cameron or foreman, but not both, is on duty and there is nobody`s shift after that
If foreman is on duty exactly thrice, and Cameron and chase are on duty exactly once, then all of the following
could be true EXCEPT?
A. Foreman is the only person on duty on Wednesday B. Cameron gets the first shift on Monday
C. Chase gets the first shift on Monday
D. All three doctor have shifts on Monday
E. None of these
9/10.
Exactly three doctors- Cameron, Foreman, and chase have been allotted shifts for their visit in the paediatric
ward on Monday, Tuesday and Wednesday -Each doctor visits at least once in the three days but never
more than once on a given day -Every day at least one doctor pays a visit to the ward and no two visit
simultaneously -On Monday foreman is on duty, and nobody has a shift after him on that day. -On

Tuesday either Cameron or chase, but not both, is on duty and there is nobody`s shift after that -On
Wednesday either Cameron or foreman, but not both, is on duty and there is nobody`s shift after that
If Cameron is on duty exactly thrice, foreman is on duty exactly twice, and chase is on duty exactly once, then
which of the following must be true?
A. All three doctor have shifts on Monday
B. Exactly two doctors have shifts on Wednesday
C. Chase and foreman both have shifts on Monday
D. Cameron is the only doctor on duty on Wednesday E. Foreman and Cameron have shifts on Tuesday
10/10.
Exactly three doctors- Cameron, Foreman, and chase have been allotted shifts for their visit in the paediatric
ward on Monday, Tuesday and Wednesday -Each doctor visits at least once in the three days but never
more than once on a given day -Every day at least one doctor pays a visit to the ward and no two visit
simultaneously -On Monday foreman is on duty, and nobody has a shift after him on that day. -On
Tuesday either Cameron or chase, but not both, is on duty and there is nobody`s shift after that -On
Wednesday either Cameron or foreman, but not both, is on duty and there is nobody`s shift after that
If chase is on duty exactly thrice, foreman is on duty exactly twice, and Cameron is on duty exactly once, then
which one of the following is a complete and accurate list of doctors who could possibly get the first
shift on Monday? A. Foreman B. Chase C. Cameron , foreman D. Cameron , chase E. Cameron ,
foreman , chase
7 Infoscions K, L, M, N, O, P and R retired over the course of 7 years, one person per year.
K retired 3 years before 0
P and M retired in consecutive years, not necessarily in the same oreder
N retired third
L did not retire sixth.
1/10. If P was not one of the first two to retire, it could be true that
A.
P retired fourth
B.
M retired fourth
C.
M retired second
D.
P cannot be retired fourth
E.
P cannot be retired seventh
2/10. Which of the following could be the order of retirement?
A.
B.
C.
D.
E.

P, M, N, K, R, L, O
M, P, N, R, K, O, L
P, N, M, R, O, K, L
M, P, N, K, L, R, O
P, M, N, O, L, K, R
3/10. If K retires fourth, which of the following must be true?

A.
B.
C.
D.
E.

P and R are retired in consecutive years


P and L are retired in consecutive years
M and R are retired in consecutive years
L and R are retired in consecutive years
M and L are retired in consecutive years
4/10. In how many different places in order could O retire?

A.
B.
C.
D.
E.

1
2
3
4
5
5/10. Which of following is possible?

A.
B.
C.
D.
E.

P retires 1 year after N


M retires 1 year after N
K retires fifth
P retires 1 year before N
P retires 4 years before O

6/10. Vikas was assigned a task to create a collection of 5 letter passwords. Each password must be
the combination 2 alphabets and three digits. Digits and alphabets need to be distinct. Find
the total number of passwords generated by Vikas.
A.
B.
C.
D.
E.

26

C2 * 10C3 * 5!
C2 * 10C3 * 5!/2!
26
C2 * 10C3 * 5!/2! * 3!
26
C2 * 10C3
None of these
26

7/10. From 6 red and 4 white balls, a collection of 5 is to be formed. In how many ways can this be
done, if the collection is to include atleast one white ball?
A.
248
B.
250
C.
246
D.
242
E.
None of these
8/10. Complete the analogy- DECK : HJFV :: FIG : ?
A.
JNJ
B.
LRM
C.
JNN
D.
LRN
E.
JRM
9/10. Find the next term
2, 1, 3, 4, 7, 11, ?
A.
18
B.
20
C.
15
D.
14
E.
4
10/10. Rohit runs 25% faster than Rajat and he is able to give Rajat a start of 10 meters to end a race together.
What is the length if the race?
A.
40m
B.
50m
C.
30m
D.
25m
E.
None of these
1/10.
Four dice are thrown simultaneously. Find the number of outcomes in which atleast one of the dice shows 3
A. 670
B. 671
C. 672
D. 673
E. 674
2/10
Three pipes A, B and C can fill a tank from empty to full in 30 minutes, 20 minutes, and 10 minutes
respectively. When the tank is empty, all three pipes are opened. A, B and C discharge chemical
solutions P, Q and R respectively. What the proportion of the solution R in the liquid in the tank after 3
minutes?
A. 6/11
B. 5/11
C. 7/11
D. 8/11
E. None of these
3/10.
There are 10 points, out of thee 10 points 4 are in a straight line and with the expectation of these four points, no
other three points are in the same straight line
Find the total number of geometrical figures formed based on below requirements:
I. The number of straight lines formed
II. The number triangles formed
III. The number of quadrilaterals formed by joining these ten lamps.

A. 171
B. 10
C. 341
D. 251
E. 291
4/10.
In a certain language, if tti wop nhi stands for he gives nothung, nsoptrklichn stands for sharma gets
marriage gift and ptrinmchn stands for wife gives marriage gift, what would mean gives?
A. Tti
B. Wop
C. Ptr
D. Chn
E. None of these
5/10
What is the 6th term in the sequence?
A3,C4,F6,J8,O12,
A. U20
B. Q21
C. U21
D. U18
E. U16
Five students-Pam, jim, Dwight, Michael, and Kevin- are punished and sent to the principals office. The
principal calls their parents. The students are sent to the detention room and made to sit in s particular
linear order. Their parents meeting order with the principal is according to some predefined order.
- Parents pf exactly one student can meet the principal at given time
- Kevin is seated on the fourth bench and his parents are the first one to visit the principal
- Michaels parents are fourth to visit
- Jim and Dwight are seated extreme ends in the detention room
- Dwights parents cannot be the second or third to meet the principal
6/10.
Who among the following could be made to sit on the third bench?
A. Student whose parents meet the principal in the first slot
B. Student whose parents meet the principal in the third slot
C. Jim
D. Dwight
E. Kevin
7/10
If fourth and third slots are assigned to the parents of the students sitting on the second and third bench
respectively, then which of the following must be true?
A. Second slot is assigned to the student sitting on the first bench
B. Pam is sitting on the third bench
C. Jim is sitting on the fifth bench
D. There is exactly one student sitting between the one whose parents are given slot 4 and the one whose parents
are given slot 5
E. There are exactly two students sitting between Jim and Michael
8/10
Each of the following can be made to sit on first and second bench, respectively, EXCEPT:
A. Jim, Pam
B. Jim, Michael
C. Dwight, Pam
D. Dwight, Michael
E. Michael, Pam
9/10
If the second slot is assigned to Pams parents and she is sitting on the second bench then, which of the
following must be true?
A. First slot is assigned to Michaels parents
B. Third slot is assigned to Jims parents
C. Fourth slot is assigned to Michaels parents and Michael is seated on the fifth bench
D. Fourth slot is assigned to kevins parents
E. Fifth slot is assigned to jims parents and Jim is seated on third bench
10/10

If pam is sitting on the second bench, then which of the following must be true?
A. The parents of the student sitting on the fifth bench meet the principal in the second slot
B. The parents of the students sitting on the third bench meet the principal in the third slot
C. The parents of the student sitting on the second bench meet the principal in the fifth slot
D. Jim is sitting on the first bench
E. Michael is sitting on the third bench
ENGLISH:
1.In a convincing test of Newtonian physics, it was anomalies in the orbit of Uranus that led astronomers to
predict discovery of Neptune.
* It was anomalies in the orbit of Uranus that
* It was the orbit of Uranus showing anomalies that
* The anomalies in the orbit of Uranus
* The orbit of Uranus being anomalous was that
2. One of the informants eventually professed ignorant of the crime, having fear that his testimony would lead to
reprisals against him by his former confederates.
* Ignorant of the crime, having
* Ignorant to the crime, with
* Ignorance of the crime, for
* Ignorance of the crime, since
3. Due to the limitations imposed by the speed of light, the visible universe is estimated as a mere fraction of the
total universe.
* Is estimated as
* Is estimated to be
* Is estimated at
* Estimated at
4. The American Civil War was ended when Lincoln was shot.
* The American Civil War has ended when Lincoln was shot.
* The American Civil War had ended when Lincoln was shot
* The American Civil War had an ended when Lincoln was shot.
* The American Civil War ended when Lincoln being shot.
5. popular child psychologists have advocated that parents discipline male children similarly to the fashion in
which they discipline daughters.
* Similarly to the fashion in which they discipline
* In the same manner that they would use with
* Like they would handle
* As they would
6. cattle were domesticated both for the uses made of the animal-food and leather-but also for the labour the
animal could provide.
* And for
* Or for
* But also
* But also for
7. Just as the sun has risen, the rooster crows.
* Even when the sun rises, the rooster crows.
* If the sun will rise, the rooster will crow.
* Just as the sun rose, the rooster crowed.
* As the sun rose, the rooster will crow.
8. several stock market analysts now reprt that the plummeting values of many high-tech stocks have fallen so as
to make them once again attractive to investors.
* Values of many high-tech stocks have fallen so as to make
* High-tech stock values have fallen, making
* Values of many high-tech stocks are making
* Values of many high-tech stocks have fallen, which has made

9. 1. Michel Sadelain is the lead scientist in the somatic cell transfer research group that was a genetic
engineering group.
2. Michel Sadelain was the lead scientist in the somatic cell transfer research group since it was a genetic
engineering group.
3. Michel Sadelain is the lead scientist in the somatic cell transfer research group, which is a genetic engineering
group.
4. Michel Sadelain is the lead scientist in the somatic cell transfer research group but it was a genetic
engineering group.
10. 1. The scientists at the Irvine Institute with the scientists at the Davis Institute could not able to crack the
viral code.
2. The scientists at the Irvine Institute or the scientists at the Davis Institute were able to crack the viral code.
3. The scientists at the Irvine Institute along with the scientists at the Davis Institute had not able to crack the
viral code.
4. Neither the scientists at the Irvine Institute nor the scientists at the Davis Institute were able to crack the viral
code.
11. 1. The nurses and the doctors take there patients case very seriously.
2. The nurses and the doctors take their patients cases very seriously.
3. The nurses and the doctors take their patients case very seriously.
4. The nurses and the doctors take their patients case very seriously.
12. 1. The survivors of the Tsunami live in a world that has been turned upside down because of all the
destruction and loss.
2. The survivors of the Tsunami living in a world that was been turned upside down because of all the
destruction and loss.
3. The survivors of the Tsunami lives in a world that has turned upside down with all the destruction and loss.
4. The survivors of the Tsunami do live in a world that has been turned upside down out because of all the
destruction and loss.
13. 1. Walking into the thicket, the nettles scratched Corbetts skin to shreds.
2. Walking into the thicket, the nettles were scratching Corbetts skin to shreds.
3. Walking into the thicket, Corbett found that the nettles scratched his skin to shreds.
4. Walking with the thicket, the nettles scratched Corbetts skin to shreds.
14. 1. For whom did Maria bake a cake?
2. For who did Maria bake a cake?
3. For what person did Maria bake a cake?
4. For which did Maria bake a cake?
15. 1. The office grapevine group specializes in talking in love affairs.
2. The office grapevine group specializes in talking about love affairs.
3. The office grapevine group specializes in talking on love affairs.
4. The office grapevine group specialized in the talking of love affairs.
16. 1. Sandra and John like to go for picnics on weekends nevertheless kea does not.
2. Sandra and John likes to go on picnics on weekends but kea does not.
3. Sandra and John like going on picnics for weekends since kea does not.
4. Although Sandra and John like to go for picnics on weekends, kea does not.
Alun Turing, the British mathematician whse concepts in the early 1950s foreshadowed the modern-day digital
computer, proposed a simple test to check for artificial intelligence. If a human judge, he said, engaged
in a natural language text conversation with two other parties, one a human and the other a machine, and
if the judge could not reliably tell which was which, then the machine would for all purposes have
passed the test.
As of 2006 no machine has managed to do that. But now a robot massager or Chabot has hit the internet. If it so
lifelike in its responses that many people have been fooled into thinking theyre talking to a human
being.
Invented by British scientists and nicknamed George, Its programmed to show emotions, tell jokes, answer
questions and engage in intimate conversation on subjects as varied as love, life and the universe.

It can also speak 40 languages as its vocabulary continues to improve which, incidentally, is bound to happen
considering George has already chatted with some two million people since its inception. George is also
capable of carrying on a conversation with hundreds of different people at the same time from all over
the globe. To some people, though, the scary part is that George continues to evolve.
From expressing itself only as a disembodied text interface, a fully-animated 3D image of an androgynouslooking humanoid capable of an extensive range of gestures and expressions has recently been
introduced to online audiences.
This new George, unlike many other controversial programs, does not merely try to be logical but attempts to
form relationships and frequently behave illogically in order to seem more alive. This is exactly what
had been predicted by people ever since the industrial revolution introduced the possibility of creating
mechanical human beings, and Mary Shelley wrote Frankestein, based on a laboratory made semihuman creature.
From expressing itself only as a disembodied text interface, a fully-animate 3D image of an androgynouslooking humanoid capable of an extensive range of gestures and expressions has recently been
introduced to online audiences.
This new George, unlike many other conversational programs, does not merely try to be logical but attempts to
form relationships and frequently behave illogically in order to seem more alive. This is exactly what
had been predicted by people ever since the industrial revolution introduced the possibility of creating
mechanical human beings, and Mary Shelley wrote Frankenstein, based on a laboratory-made semihuman creature.
The paranola generated by such things has only gathered momentum since then, so that today with the advent of
powerful computers it has lead to an overwhelming feeling of fear of a complete takeover by machines
in the future.
The reason is that robotic creatures can now also be imbued with artificial intelligence which rivals that of
human beings. When robots supersede human beings there would no longer be any need for the Turning
Test, unless of course, machines start testing us for intelligence instead.
17. What is the authors greatest fear?
* Humans will be rivaled by machines
* Machines ruling us in the future
* Robots passing the Turning test
* Machines testing us for intelligence
18. As understood from the passage, what makes George seem more life-like?
* Extensive range of gestures and expressions
* Attempts to form relationships
* Ability to talk on varied subjects
* Illogical behaviour
19. According to the passage, what was it that no machine had managed to do?
* Pass the Turning test
* Learn many languages
* Participate in conversation
* Show emotions and make gestures
20. The robot massager George was invented by
* Alan Turing, the British mathematician
* American scientists
* Scientists from Britain
* Mary Shelley
21. What was the impetus for the new robot?
* Mary Shelleys book Frankenstein
* Industrial revolution which saw the possibility of making mechanical human-beings
* Laboratory-made semi humans.
* All of the above
22. Drug addiction and alcohol abuse are not something totally new in Indian society. They have been there for
quite some time and ..................... today.
* Are problems that persists even
* Shows persistence even
* Was a problem that persists even
* Is a persisting problem of
23. Consolidation of knowledge in the field of cryogenics ............ cryobiology and medicine.

* Will have broadened our understanding on


* Has broadened a understanding of
* Is broadening our understanding to
* Has broadened our understanding of
24. Jones was admitted to the hospital with sharp chest and shoulder pain. Though the ECG was normal, the
doctors concluded that he .................
* Had an heart attack.
* Is having a heart attack.
* Must have had a heart attack.
* May have had an heart attack.
25. .................., Blakes work, produced in partnership with his wife Catherine, is widely known today.
* Though largely unrecognised during his lifetime
* Neither largely unrecognised during his lifetime
* Although largely unrecognised between his lifetime.
* However largely unrecognised in his lifetime.
26. Nobody can help a person come out of depression and gloom. ............................... or take the help of
psychiatrists and medicines.
* Either the person should have will power
* So the person should have will power
* But the person must have will power
* Neither the person should have will power
27. The meaning of the word raconteur is .................
* A furry little animal.
* A gossip monger.
* An inveterate chatter box.
* A person skilled in telling anecdotes.
28. The math tutor said that his students ......... their basics and then go on to complex problem solving.
* Should brush off
* Has to brush with
* Should brush up on
* Could brush up in
29. After playing tennis in the hot sun, Sania came inside and .......................towards the cold water bottles in
the fridge.
* Made a gold rush
* Made a beeline
* Took a dive
* Made a clean sweep
Passage:
Take the four-lane bypass near Moradabad that will one day merge with the Grand Quadrilateral, a stretch of tar
that curves and caresses the ground like any American expressway. For a few moments you will be
lulled into the belief that you are on Route 95 connecting New York and Washington.
Raised above natural ground and protected by steel guardrails, the noiseless acceleration and absence of
roadside stalls leaves you wondering where you are, a little uncomfortable, a little lost. The permanent
markers of India, the persistence of its urban topography, is lost in the monochromatic gaze that encloses
nothing, nothing but a stretch of road and horizon.
It is easy to be seduced by the engineering specifically of the landscape. But where are the people who normally
make a living off the road? Where are those who live off the land? Where in hell is India in the picture?
The simple truth is that India is not in the picture. IUn the grand scheme of globalisation, airport upgradation,
French wineries, German chocolate and Thai restaurants, heritage spas, infotech cities, Hyundai
factories, Amby valleys and heritage hotels, baroque houses, Brazilian vacations, BMW showrooms and
LOreal fashion parades, the people of Moradabad do not matter. When the goodies of globalisation
were being doled out, the India behind the Golden Quadrilateral just got left out. Every few weeks I
travel the same road across Moradabad on the way to a construction site. I have travelled on the same
stretch of road for the past 16 years. In that time, when economic liberalisation, free trade and
consumerism crept into Indian politics, little along the 150 kilometre stretch changed. The milk booth
became a beer bar, the dhaba owner added chow mein to the menu, the man at the chana cart is older, the
pi-dogs fatter and healthier, garbage and waste more visible.
Turn off the Moradabad bypass, and old India envelops you like a dust storm in summer. The Golden
Quadrilateral becomes a dirt track; a lone buffalo drags the wooden wheels of a cart at 18th century
speed.

Mile upon relentless mile, the scene refuses to change: A tree in the distance, a mud hut, stick and charpoy, a
primitive land, now saddled with 21st century problems of pollution, deforestation, and soil erosion.
Across dry sun-burnt ground it is hard to imagine a ore inhospitab;e place; yet people live, carrying water long
distances, scrounging the ground for firewood, burning each other for using the wrong well or marrying
outside their castes.
In the middle of this emptiness, driving through a dust haze, I once came across an old woman sitting by the side
of the track with an insignificant heap of garlic, maybe 20-25 cloves. Sitting cross-legged and staring
into nothing. On my return, she would still be there with the same heap, leaving me thinking all the way
home of our acceptance of such astonishing differences and disparities.
Each day, you are reminded of your place in India. Your difference from the woman selling the garlic, the man
on the street, the family in the hotel pool, the ragged mali watching them. India reminds you of its fullblown deprivations, by also giving you constant reminders of its opportunities. In your sightline is
someone worthy of emulation, as well as someone in desperation. Contradictory signs are everywhere,
yet they baffle no one. Daily collisions across social political and religious boundaries occur everyday.
In the blinkered Indian view, out of the tinted windows of a car, they are but disjointed scenes from a play. What
appears sad and funny and stupid and tragic is the irony of the juxtapositions. BMW, one of the worlds
fastest machines, drives on a rutted village road; the international water theme park is built along a
rivulet of raw sewage, and often in an area of drought; the industrialists road; the international water
theme park is built along a rivulet of raw sewage, and often in an area of drought; the industrialists high
profits are made at the expense of people whose daily wage is less than the cost of pissing in a London
pay-toilet.
From a narrow middle-class perspective, buttressed by privilege, protected and cocooned by inheritance, such
contrasts become theatre. India of the 21st century belongs only to those living on the two extremes.
For the rising tide of call centre operators, entrepreneurs and high stock industrialists at one end, India is instant
opportunity. For the silent majority, farmers, stone cutters and child labourers, India is a perpetual
struggle. For the middle class, both Indias are an exaggeration. In every society, it is the middle class
that bears the burden of dysfunction. Those of my background, privileged and pretentious, a little aloof,
and always judgmental, are easily unhinged in an untamed place.
Without the daily crossfire of survival or aspiration to occupy you, India is an adequate ally, just an indulgence.
A great peep show; I will pass the garlic-seller woman again. And continue to be appalled.
30. Why does the author state the example of the garlic seller?
* To show the state of her poverty
* To say how things have remained static for some people
* To strike a difference between the rich and the poor
* To highlight the apathy of the people around
31. Who according to the author is the most affected by the vast differences between the two classes?
* The politicians
* The social reformers
* The industrialists
* The middle class.
32. According to the passage, which of the following changes has the author not seen in six teen years?
* The real effect of economic liberalization
* Beer bars springing up on the highway
* International cuisine on the menu in dhabas
* Strays growing healthier.
33. In the passage the author is primarily concerned with
* Showcasing the poverty in India
* Comparing development in India with that of America
* Bringing out the stark contrast between the affluent and the deprived.
* Glorifying the economy that has seen a significant improvement.
34. The word lulled is closest in meaning to
* Tricked
* Deceived
* Interrupted
* Tranquilized
Select the best of the answer choices given

35. To understand the characteristics of learners, educational psychology develops and applies theories of
human development. At present, educational psychologists at Marvin High School in the United
Kingdom are trying to analyze how and why their students acquire an aversion towards math.
Which of the following statements is a false assumption?
* Educational psychology aims at trying to make understand the process of learning.
* Educational psychology is restricted only to learners in formal educational institutions.
* Learning is a key part of human development
* Marvin High School is taking the help of educational psychologists to solve their students learning problem.
36. Medical logistics is the logistics of pharmaceuticals, medical and surgical supplies, medical devices and
equipment, and other products needed to support doctors, nurses, and other health and dental care
providers. While business logistics aims to maximize efficiency and increase savings, medical logistics
aims at maximizing effectiveness. Therefore, medical logistics is quite different from general logistics.
Which of the following statements will strengthen the above argument?
* Because its final customers are responsible for the lives and health of their patients, medical logistics has to
focus on effectiveness rather than profit.
* Medical logistics companies always run under a loss.
* Without proper profits, medical logistics companies cannot operate effectively.
* Business logistics is inhuman and cut throat.
37. Geriatrics is the branch of medicine that focuses on health promotion and the prevention and treatment of
disease and disability in later life. Most hospitals in advanced countries have a special geriatrics
developed with specialised doctors and caregivers. A statistics survey across Europe and Asia showed
that elders in Europe are healthier and happier than their Asian counterparts.
Which of the following can be inferred from above?
* Geriatrics can be life saving for old people.
* People live longer in cold countries.
* Diseases accompanying old age are eliminated by European geriatric specialists.
* Longevity is possible if specialised care is given to elders.
38. Statesmen in the United States of America have to be graduates in order to contest for elections. Take the
example of Bobby Jindal who has a Bachelors Degree in Business Studies. Coming to our own country,
we have Minister Akela Singh who is an illiterate and cannot even sign his own name.
Which of the following statements can be inferred to be true?
* Akela Singh is not a statesman
* Bobby Jindal is not a statesman
* Our country does not have a minimum qualification for entry to politics.
* None of he above.
39. Law is a set of rules or norms of conduct that forbids, permits or mandates specified actions and
relationships among people and organisations. Islamic Law is different from Hindu Law as is Jewish
Law different from Native American Law.
Which of the following statements is a logical conclusion to the above passage?
* Law is not universal; some societies do not have laws.
* Some laws do not permit close friendships in society.
* If A is a criminal in Islamic society, he/she she is not a criminal in Native American Society.
* The legal system, though based on certain commonalities, varies according to the law making bodies in
different societies.
40. In Axis Company, most of the employees are having bouts of back pain, shoulder pain, and neck pain. Most
of them are not comfortable while working. The company doctor referred them to the work-related stress
injuries department in an orthopaedic hospital.
Which of the following statements can be inferred from the above passage?
* Axis company needs to make ergonomic seating arrangements for its employees.
* Most of the employees are weaklings.
* The company doctor gets a cut for every patient referred to the orthopedic hospital.
* Axis Company does not take care of its employees
Directions: For each question in this section, select the best of the answer choices given
1 / 40. After years of status as the far behind No. 5 to the Big Four Indian software companies, Shipra
Software is finally catching up. it will hit St billion In revenues this year, thanks to expansion
into new areas such as infrastructure-management and software -for engineering processes In
the auto and defense industries. That's in addition to the application-maintenance work the
company ran on for years. Shipra's deals are getting bigger, too.
Which of the following statements, if true, will check Shipra's growth in the software chart?
If Shipra Software Is catching up in the race, the companies ranked higher than Shipra are aiming higher
and their deals are growing too.
Shipra Software is going to be taken over by an WIC which has already acquired 90% of Shipras shares.
The software bubble has just gone bust and Shipra's deals are off.

None of the above.


2 / 40. Film is considered by many to be an important art form; films entertain, educate, enlighten and inspire
audiences. The visual elements of cinema need no translation. German movies are watched by Indians,
Hindi movies are watched by the British, French movies are watched by the Russians. which of the
following can be inferred from the above?
English movies are no longer preferred by viewers.
People have become proficient in more than one foreign language.
Motion pictures have the universal power of communication.
The British have happily embraced Indian culture.
3 / 40. When the first American music videos and popular TY shows began appearing in Indian homes
in the early 1990s thanks to satellite and cable, many pundits predicted Indian society would
never be the same. Indeed, the impact on younger generations of Indian women has been
profound. Indian women traditionally have been submissive to parents and husbands and valued
frugality and modesty in the past.
Which of the following is a false assumption?
Indian women are trying to catch up with their western counterparts.
Indian women will come out of the confines of the household and participate in public Life.
Indian women are going to a life of wild abandon and splurge their money.
Indian women have been watching Western TV channels and shows.
4 / 40. The Axis Company Is a young and energetic one trying to set new standards in work practices
and ethics. The management is trying to formulate new policies to make the work culture
friendly and informal. One of the policies formulated by the management is, You are
accountable for your own behaviour. You are answerable only to yourself."
Which of the following statement is a direct contrast to the above?
You have to report everything to your immediate supervisor.
You have to answer for your team.
A team comprising whistlebtowers has been set up.
You have to explain your performance.
5 / 40. Acquired immunodeficiency syndrome or Acquired immune deficiency syndrome (AIDS or
Aids) is a collection of symptoms and infections in humans resulting from the specific damage
to the immune system. A human immunodeficiency virus (NW) test cannot determine if a
person has full-blown AIDS though AIDS is caused by HIV.
Which of the following statements can be inferred from the above passage?

HIV alone does not destroy the Immune system.


There is no proper laboratory test for AIDS.
A person who is infected with HIV does not necessarily have AIDS.
Though HIV causes AIDS. Other viruses enter the system and cause different illnesses.
6 / 40. Currently, in all countries with a democratic system and the rule of Law, criminal procedure puts
the burden of proof on the prosecution - that is, it is up to the prosecution to prove that the
defendant is guilty, as opposed to having the defendant prove that he is innocent: any doubt is
resolved in favour of the defendant.
which of the following statements Is a logical conclusion to the above passage?
No wonder. Indian criminals get away scot-free.
Democratic legal systems include a provision known as the presumption of innocence.
Democracy is soft on criminals.
Democracy is tough on the Judicial System.
Questions: Identify the correct sentence
7 / 40. 1. AS he was once an avid soccer fan. he is now increasingly becoming interested in cricket.
2. Once an avid soccer fan, he is now becoming more interested in cricket.
3. Being once an avid soccer fan, he is now increasingly becoming a cricket fan.
4. Now he is becoming interested in cricket but was an avid soccer fan once before.

8 /40. 1. Broadly speaking. curriculum Includes all experiences which the student may have within the
environment of the school.
2. To speak broadly, curriculum includes alt experiences which the student may have within the
environment of the school.
3. Broad speaking, curriculum means all the excriences of the student in the school environment.
4. All the experiences of the student in the school environment are curriculum to speak broadly.
9 / 40. 1. Although most rocks contain several minerals, limestone contains only one and also marble
too.
2. Although most rocks contain several minerals, limestone contains only one and so does marble.
3. Although most rocks contain several minerals, limestone contains one only and also marble.
4. Although most rocks contain several minerals, limestone contains one and marble also.
10 / 40. 1. Tourists are able to dearly see the City of New York on top of the Empire State Building.
2. From the Empire State Building, tourists are able to dearly see the City of New York.
3. Tourists are able to see the City of New York dearly from the top of the Empire State Building.
4. From the top of the Empire State Building, the City of New York was dearly seen by the tourists.
11 / 40. 1. The Federal Trade Commission may intervene whenever unfair business practices,
particularly monopolies, are suspected.
2. The Federal Trade Commission may intervene whenever business practices are unfair and
particularly monopolies are suspected.
3. The Federal Trade Commission may have to intervene whenever they suspect unfair
monopolies and business practices.
4. The Federal Trade Commission may intervene wherever they suspect monopolies and
business practices.
12 / 40. 1. To judge your friends, you should not listen to what they say only observe what they do.
2. To Judge your friends, you should not only listen to what they say but also observe what they do.
3. To be able to judge your friends, you just should not listen to what they say and observe what they do.
4. To judge your friends you only observe what they do and not what they say.
13 / 40. 1. Adolescence, or the transitional period between childhood and adulthood is not only a
biological concept but a social concept.
2. Adolescence, or the transitional period from childhood to adulthood, is not only a biological concept
but a social one too.
3. Adolescence, or the transitional period between childhood and adulthood is not just a biological
concept but a social concept also.
4. Adolescence, or the transitional period between childhood and adulthood, is not just a biological
concept but a social one as well.
14 / 40. 1. The bill which we received is higher than the estimate.
2. The bill which we received was higher than the estimate.
3. The bill that we received is more higher than the estimate.
4. The bill that we had received is higher than they had estimated.
Directions: Fill in the blank with the correct option that fits in grammatically and logically.
15 / 40. A new automobile needs to be after the first five thousand mikes.
tuned in
tuned out
tuned up
tuned down
16 / 40. I have a feeling inside me that what she said was a lie. It lust
was a likely story.
rang a bell.
didn't ring true.
drew a red herring.
17 / 40. Aristotle is the father of logic.
Commemorated as
Felt to be
Recognized as
Counted as
18 / 40. I've heard enough of pop music; Im tired___________it.

of hearing
to hearing
to hear
of listening
19 / 40. It is not the examination but rather the academic preparation of a student that is the best
indicator
to his success
of his success
for his success
for him to succeed
20 / 40. A great many athletes have managed to_____________serious physical handicaps.
overlook
overbear
overthrow
overcome
21 / 40. The government always tries to____________when there are economic problems. Saying the
previous regime is to blame.
start the ball rolling
take the plunge
go back to square one
pass the buck
22 / 40. Giving severe punishment will set an example to those who have the propensity____________
because of their money power, social status and posts held.
To disregard court orders
to dispense with court orders
to deal with court orders
to depreciate court orders
Directions: Read the following passage below and answer the questions that follow on the basis of what
is stated / Implied in that passage.
Man is the central mystery of the universe holding the key to all other mysteries. Indeed, human beings
are our own greatest enigma. As the famous physicist Niels Bohr once said, 'Me are both
spectators and actors in the great drama of existence.' Hence the Importance of developing of
what is known as the "science of human possibilities: It was such a science that India sought and
found in the Upanishads in an attempt to unravel the mystery of man.
Today we see a growing urge in everyone to realize the 'true self. We are keenly feeling the need to
make our knowledge flower into wisdom. A strange yearning to know about the infinite and the
eternal disturbs us. It is against this background of modern thought and aspirations that the
contributions of the Upanishads to the human cultural legacy become significant
The purpose of the Vedas was to ensure the true welfare of all beings, wordly as well as spiritually.
Before such a synthesis could be achieved, there was a need to penetrate the inner worlds to its
depth. This is what the Upanishads did with precision and gave us the science of the self, which
helps man leave behind the body, the senses, the ego and all other non-self-elements, which are
perishable. The Upanishads tell us the great saga of this discovery of the divine in the heart
of man.
Very early in the development of the Indian civilization, man became aware of a strange new field of
human experience the within of nature as revealed in man, and in his consciousness and his
ego. It gathered volume and power as years rolled on until in the Upanishads it became a deluge
issuing in a systematic, objective and scientific pursuit of truth in the depth of experience. It
conveys to us an Impression of the tremendous fascination that this new field of Inquiry held for
the contemporary mind.
These Indian thinkers were not satisfied with their intellectual speculations. They discovered that the
universe remained a mystery and the mystery only deepened with the advance of such
knowledge, and one of the important components of that deepening mystery is the mystery of
man himself. The Upanishads became aware of this truth, which modem science now
emphasizes.
23 / 40. In the phrase it became a deluge, in the passage, it refers to
Indian civilization
Upanishads
man's innermost self

Human experience
24 / 40. Man is considered an enigma because
he is both protagonist and spectator.
he holds the key to the mysteries of the universe.
he feels the need to turn his knowledge into wisdom.
he experiences the conflict between the material and the spiritual world.
25 / 40. The Upanishads have become significant today as:
Indians want to prove to the world that the secret of the self is hidden in the Upanishads
man is aspiring to unravel the truth of the self
we want to make our knowledge flower into wisdom
we are disturbed by the concept of the eternal
26 / 40. The greatest mystery of the universe is
Upanishads
ancient civilization
man
vedas
27 / 40. The Upanishads
offer solutions to the mystery of the universe
complement science in its pursuit of truth
try to provide insight into the divinity of man
are a prelude to the Vedas
Directions: Read the following passage below and answer the questions that follow on the basis of what
is stated / implied in that passage.
The Great White Shark (Carcharodoncarcharias), also known as white Pointer, White Shark, or White
Death is an exceptionally large lamniform shark found in coastal surface waters in all major
oceans. Reaching lengths of about 6 meters (20 feet) and weighing almost 2.000 kilograms
(4,000 pounds). the Great white is the worlds largest predatory fish. They are the only known
surviving species of their genus, Carcharodon.
Great white Sharks live in almost all the cold or temperate waters of the planet. with greater
concentrations in the southern coasts of Australia, in South Africa. California. and to a degree in
the Central mediterranean and the Adriatic Sea. The densest known population is found around
Dyer Island, South Africa where up to 31 different white sharks have been documented by
Michael Scholl of White Shark Trust in a single day. It can be also found in tropical waters like
those of the Caribbean and has been recorded off mauritius. It is also a pelagic fish. but recorded
or observed mostly in coastal waters in the presence of rich game like otariids, cetaceans. other
sharks and large bony fish species. It is cprisidered an open-ocean dweller and is found from the
surface to as deep as 1280 meters. but is most often seen near the surface:
The Great white Shark has a robust large conical-shaped snout. It has almost the same size upper and
lower lobes on the tail fin (like most mackerel shark, but unlike most other sharks). It is pale to
dark gray and has a white belly. Great whites. like many other sharks. have rows of teeth behind
the main ones, allowing any that break off to be rapidly replaced. Their teeth are unattathed to
the jaw and are retractable, like a cat's claws. moving into place when the jaw is opened. Their
teeth also rotate on their own axis (outward when the jaw is opened. inward when closed). The
teeth are linked to pressure and tensor-sensing nerve cells. This arrangement seems to give their
teeth high tactile sensitivity.
white sharks' reputation as ferocious predators is well-earned. yet they are not (as was once believed)
indiscriminate "eating machines'. They typically hunt using an 'ambush' technique. taking their
prey by surprise from the bottom. Sometime they swim so fast that they actually jump out of the
water while chasingtattacking seals. This is one of only a few sharks that can jump fully out.of
the water, the others are Thresher shark, Shortfinmako, Longf in mako, Spinner shark, BlaThtip
reef shark, Salmon shark, Porbeagle shark and the Copper shark. This is the only shark known
to regularly lift its head above the sea surface to gaze at other objects such as prey; this Is
known as "spy-hopping", this behaviour have also been seen in at least one group of Bladdip
reef sharks but this might be a behaviour learned from Interaction with humans. It is theorized
that the shark may also be able to smell better this way, since smells travel through air faster
than through water.

More than any documented attack, Steven Spielberg's 1975 film Jaws provided the Great White with the
image of a "man eater' in the public mind. While Great Whites have been responsible for
occasional fatalities in humans, they typically do not target humans as prey: for example, in the
Mediterranean Sea there were at confirmed attacks against humans in the last two centuries,
only a small number of them deadly. marry incidents seem to be caused by the animals 'testbiting" out of curiosity. Great White Sharks are known to perform test-biting with buoys,
flotsam, and other unfamiliar objects as welt, and might grab a human or a surfboard with their
mouth (their only tactile organ) in order to determine what kind of object it might be.
28 / 40. what arrangement gives a shark's teeth high tactile sensitivity?
A. Their teeth are unattached to the jaw and are retractable.
B. Their teeth also rotate on their own axis.
C. The teeth are linked to pressure and tensor-sensing nerve cells.
A only
B only
A, B and C
B and C
29 / 40. why Is the the Great white Shark is considered an open-ocean dweller?
Great White Sharks live in almost all the cold or temperate waters.
Great White Sharks are observed mostly in coastal waters.
It can be also found in tropical waters.
Great White Sharks are found near the surface as well as in the depths of the ocean.
30 / 40. Which behaviour is learnt by the shark due to human-shark interactions?
Sharks are sometimes known to regularly lift its head above the sea surface to gaze at other objects such
as prey.
Sharks typically hunt using an "ambush' technique.
Sharks can jump fully out of the water.
It is theorized that the shark may also be able to smell better.

31 / 40. what characteristic of the Great White Shark can be Inferred from the tag, White Death?
The Great white Shark is known to attack and kill humans.
The Great White Shark does not spare smaller fishes.
The Great white Shark has teeth like sharp knives.
The Great White Shark is a man eater and thrives on human flesh.
32/40. why is Steven Spie'berg's 1975 film .taws a wrong portrayal of the behavior of sharks?
Only a small number of sharks are deadly.
Most incidents of shark bites seem to be caused by the sharks 'test-biting" out of curiosity.
Sharks typically do not target humans as prey.
Great White Sharks are deliberately provoked with buoys, flotsam, and other unfamiliar objects.
Directions: In each of the following sentences, some part of the sentence or the entire sentence is
underlined. Beneath each sentence you will find four ways of phrasing the underlined part This
Is a test of correctness and effectiveness of expression. In choosing answers, follow the
requirements of standard written English, that Is, pay attention to grammar, choice of words, and
sentence construction. Choose the answer that expresses most effectively what is presented in
the original sentence; this answer should be dear and exact, without awkwardness, ambiguity or
redundancy.
33 /40. According to the United States Bureau of Labor Statistics more than 60 percent of the workplace
Illnesses reported on a yearly basis are being associated with repetitive stress injuries (RSI).
more than 60 percent of the workplace illnesses reported In each year are associating to
morethen 60 percent of the workplace illnesses reported each year are associated to
more than 60 percent of the workplace illnesses reported each year is associated with
most of the 60 percent workplace Illnesses reported each year are associated with
34 / 40. Students of jurisprudence aim to understand the fundamental nature of law, and to analyze their
purpose, structure, and application.
toanalyze its purpose, to analyze its structure, and also to analyze its application
toanalyze its purpose, structure, and application

for analysis of purpose, structure, and application


toanalyzeit's purpose, structure, and application
35 / 40. A combat engineer is a military specialist in using the tools and techniques of engineering under
combat conditions which may perform any of a variety of tasks.
who could be able to perform any of the variety of tasks.
who may perform many of a variety of tasks.
what may perform any of a variety of tasks.
who may perform any of a variety of tasks.
36 / 40. In ancient times, the most common form in advertising was by word of mouth however,
commercial messages and election campaign displays have been found in the ruins of Pompeii.
the most common form of advertising was through words of mouth
a most common form of advertising was by the word of mouth
thecommonest form of advertising were by word of mouth
the most common form of advertising was by word of mouth
37 / 40. The modem criminal justice system was evolved since ancient times, with new forms of
punishment, added rights for offenders and victims, and policing reforms.
has evolved since ancient times, for new forms In punishment
has been evolved since ancient times, with new forms of punishment
has evolved since ancient times, with new forms of punishment
has evolved for ancient times, with new forms of punishment
38 / 40. Physiotherapists' scope of practice varied considerably across the world, both in terms of the
degree of professional autonomy enjoyed and the range of conditions managed.
Physiotherapists scope of practice varies considerably across the world
Physiotherapists scope of practice varies considerably In the world
Physiotherapists' scope of practice Is varied considerably across the world
Physiotherapists' scope of practice Is being varied considerably throughout the world
39 / 40. Research gerontologists conduct research into the aging process and the living environments in
older persons in an effort to understand and enrich the lives of elders.
on the aging process and the living environments of older persons
in the aging process and the live environment of older persons
on the aging process and the life environments of elder persons
for the age process and the living environments of elderly persons
40 40. Between the extremes by optimized health and death from starvation or malnutrition, there is an
array of disease states that can be caused or alleviated by changes in diet.
Beyond the extremes of optimal health and death from starvation
Between the extremes of optimal health and death from starvation
Behind the extremes of optimal health and death from starvation
Between the extremes in optimal health and death by starvation
Questions: identify the correct sentences:
1/40
Questions: identify the correct sentences:
A. The city Rehabilitation Center trains the mentally challenged to take care on their own
B. The city Rehabilitation Center trains the mentally challenged to take care of their own selves
C. The city Rehabilitation Center trains the mentally challenged to take care of himself
D. The city Rehabilitation Center trains the mentally challenged to take care of themselves
2/40.
Questions: identify the correct sentences:
A. The little puppy sitting on top of the wall is always seeming playful
B. The little puppy sitting on top of the wall is always seemed playful
C. The little puppy that was sitting on top of the wall always seemed playful
D. The little puppy sitting on top of the wall will always seem playful
3/40.
A. Helen keller`s nurse taught her to read, write, and use a typewriter
B. Helen keller`s nurse taught her to read, write, and typing
C. Helen keller`s nurse taught her to reading, writing, and use a typewriter
D. Helen keller`s nurse taught her to read, to write, and use a typewriter
4/40.
A. Each one of Harold Pinter`s plays used the philosophy of existentialism
B. Each one of Harold Pinter`s plays uses the philosophy of existentialism

C. Each one of Harold Pinter`s plays use the philosophy of existentialism


D. Each one of Harold Pinter`s play is using the philosophy of existentialism
5/40.
A. One of the hallmarks of miser is sponging on other peoples` money
B. A hallmark of the miser is sponging into other people`s money
C. One of the hallmarks of a miser is sponging on other people`s money
D. One of the hallmark of miser is sponging on other people`s money
6/40.
A. A boastful show off is full of sound and fury and usually has nothing interesting to talk about
B. A boastful show off is full of sound and fury and usually has anything interesting to talk about
C. A boastful show off is full of sound and fury and usually had something interesting to talk about
D. A boastful show off is full of sound and fury and usually had a few things interesting to talk about
7/40
A. The doctor seriously advised the patient and told him that he should be sleeping enough, that he should be
drinking a lot of fluids and do some exercises
B. The doctor seriously advised the patient and told him that he should sleep enough, that he should be drinking
a lot of fluids and doing some exercises
C. The doctor seriously advised the patient and told him that he should sleep enough, that he should take a lot of
fluids, and do some exercises
D. The doctor seriously advised the patient and told him that he should sleep enough, take a lot of fluids and do
some exercises
8/40.
A. If the company`s productivity do not improve, lay-offs and retrenchment can be expected soon
B. If the company`s productivity is not improving, lay-offs and retrenchment can be expected soon
C. If the company`s productivity does not improve, lay-offs and retrenchment can be expected soon
D. If the company`s productivity should not improve, lay-offs and retrenchment can be expected soon
9/40.
Directions: Read the following passage below and the answer the questions that follow on the basis of what is
stated/implied in that passage.
Man is the central mystery of the universe holding the key to all other mysteries. Indeed , human beings are our
own greatest enigma. As the famous physicist Niel`s Bohr once said, `we are both spectators and actors
in the great drama of existence. Hence the importance of developing of what is known as the `science of
human possibilities.` It was such a science that India sought and found in the Upanishads in an attempt
to unravel the mystery of man. Today we see a growing urge in everyone to realize the `trueself`. We are
keenly feeling the need to make our knowledge flower into the wisdom. A strange yearning to know
about the infinite and the disturbs us. It is against this background of modern thought and aspirations
that the contributions of the Upanishads to the human cultural legacy become significant. The purpose of
the vedas was to ensure the true welfare of all beings, wordly as well as spiritually. Before such a
synthesis could be achieved, there was a need to penetrate the inner words to its depth. This is what the
Upanishads did with precision and gave us the science of the self, which helps man leave behind the
body, the senses, the ego and all other non-self elements, which are perishable. The Upanishads tell us
the great saga of this discovery of the divine in the heart of man. Very early in the development of the
Indian civilization, man became aware of a strange new field of human experience the within of nature
as revealed in man, and in his consciousness and his ego. It gathered volume and power as years rolled
on until in the Upanishads It became a deluge issuing in a systematic, objective and the scientific pursuit
of truth in the depth of experience. It conveys to us an impression of the tremendous fascination that this
new field of inquiry held for the contemporary mind. These Indian thinkers were not satisfied with their
intellectual speculations. The discovered that the universe remained a mystery and the mystery only
deepened with the advance of such knowledge , and one of the important components of that deepening
mystery is the mystery of man himself. The Upanishads became aware of this truth, which modern
science now emphasizes.
The Upanishads have become significant today as:
A. Indians want to prove to the world that the secrete of the self is hidden in the Upanishads
B. Man Is aspiring to unravel the truth of the self
C. We want to make our knowledge flower into wisdom
D. We are disturbed by the concept of the eternal
10/40.

In the phrase it became a deluge, in the passage, `it` refers to


A. Indian civilization
B. Upanishads
C. Man`s innermost self
D. Human experience
11/40.
The Upanishads
A. Offer solutions to the mystery of the universe
B. Complement science in its pursuit of truth
C. Try to provide insight into the divinity of man
D. Are a prelude to the Vedas
12/40.
The greatest mystery of the universe is
A. Upanishads
B. Ancient civilization
C. Man
D. Vedas
13/40.
Man is considered an enigma because
A. He is both protagonist and spectator
B. He holds the key to the mysteries of the universe
C. He feels the need to turn his knowledge into wisdom
D. He experiences the conflict between the material and the spiritual world
14/40. Directions: Read the following passage below and the answer the questions that follow on the basis of
what is stated/implied in that passage. Every channel was showing the same pictures: one of the towers
belonging to the World Trade Centre in flames, the next plane approaching, a new fire and the collapse
of both buildings: the disaster of 11th September 2001. No one will ever forget where they were, what
they were doing and who they were with when the terrorist attack occurred. It is always very hard to
accept that a tragedy can, in some way, have positive results. As we gazed in horror at what looked more
like a scene from a science fiction movie the two towers crumbling and carrying thousands of people
with them as they fell we had two immediate responses: first, a scene of impotence and terror in the
face of what was happening; second , a sense that the world would never be the same again. It was with
these feelings in my heart that I switched off the TV and headed for the bookstore where the booksigning was, in theory, to take place. I was convinced that no one should be there, since the next few
hours were bound to be taken up with more news and more details and with the search for reasons why
it had happened. I walked the deserted streets of munich. Even though it was still only four o`clock in
the afternoon, people had congregated in any bars that had radios and televisions turned on, trying to
persuade themselves that it was all some kind of dream from this kind of recurring nightmare. To my
surprise , when I got to the bookshop, hundreds of readers were waiting for me. They weren`t talking to
each other, they were silent it was a silence that came from the depths of their souls, a silence empty of
meanings. Gradually, I understood why they were there : at such times, it is good to be with other
people, because no one knows what might happen next. Gradually , we all came to realize that it wasn`t
a nightmare, but something real and palpable, which from then on, would become part of the history of
our civilization. That is what I would like to write about at the end of this year of upheavals. The world
will never be the same, it`s true, but, a year on from that afternoon is there still a sense that all those
people died in vain? or can something other than death, dust and twisted steel be found beneath the
rubble of the rubble of the World Trade Center? I believe that the life of every human being is, at some
point, touched by tragedy; it could be the destruction of a city, the death of a child, a baseless accusation,
an illness that appears without warning and brings with it permanent disability. Life is a constant risk,
and anyone who forgets will be unprepared for the challenges that fate may have in store for us.
Whenever we come face to face with that inevitable suffering, we are forced to try and make some sense
of what is happening. However good we are, however well we try to live our lives, tragedies happen. We
can find some justification or imagine how different lives would have been without them, but none of
those matters : the tragedy has happened, and that`s that. From then on, what we have to do is to look
again at our lives, overcome our fear and begin a process of reconstruction.
Which of the following may be a reasonable inference from your understanding of the passage?
A. The disaster of September 11, 2001 created a sensation among the public
B. Such terrorist activities are inevitable in the modern world
C. The author is a Swiss in nationality
D. Even in the face of the worst tragedies, we should never despair but show resilience to get on with life

15/40. According to the passage


A. The book store was empty
B. People preferred to be in the company of others in the face of disasters
C. People wanted to believe that whatever they saw on the TV channels were just part of a nightmare
A. A only
B. A & B
C. B & C
D. A, B, & C
16/40. Which of the following statements is the author most unlikely to agree with? A human being encounters a
tragedy at some point or the other in his life
A. Anyone who forgets that life is at constant risk will be unprepared to meet the challenges that fate poses
B. The streets of Munich were desolate as early as 4 O` clock
C. It is unrealistic to expect people to begin the process of reconstruction and get on with their lives in the face
of adversities
D. None of these
17/40. What according to the author was like a scene from science fiction movie?
A. A silence that emanated from the depths of the souls that were hurt
B. The crumbling down of the twin towers
C. The inevitable suffering of those people in the bookstore
D. The television program
18/40.
It can be inferred about the attitude of the author that he is
A. Skeptical
B. Pessimistic C. Indignant
D. Optimistic
Directions: Fill in the blank with the correct option that fits in grammatically and logically
19/40.
Directions: Fill in the blank with the correct option that fits in grammatically and logically ______________,
I don`t mind which hotel we stay at
A. Provided the room is clean B. Lest the room is clean C. Providing the room clean D. Except the room is clean
20/40.
If you are ______________ with this test, you should perhaps attempt an easier one
A. Stumbling B. Stressed C. Straining D. Struggling
21/40.
Production this year is ________ production last year
A. Six times as greater than B. Six times as higher as C. Six times as much as D. Six times more higher than
22/40.
Some old houses were __________ to make way for the new shopping mall
A. Knocked about B. Knocked off C. Knocked down D. Knocked out
23/40. They`ve been _________ out repairs on the bridge for a long time
A. Carrying B. Bringing C. Doing D. Sending
24/40. I think we`ll have to ________ off our meeting this afternoon

A. Cut B. Call C. Set D. Cancel

25/40. _____________ sold our stocks when the markets crashed. A. If only we had B. Just we had C. Hardly
had we D. In as much as we had
26/40.
I did German at school, but _____________ most of it now
A. Forgot B. I have forgotten C. Would have forgotten D. Had forgotten
27/40. The Mughal emperor Aurangzeb observed certain rules in his delay prayers. He had to face mecca like all
Muslims do. During his morning prayer he was particular that the sun should not be in front of him or
behind him. The sun should not be to the north east, east, or west of him. Which side will Aurangzeb
face when doing his morning prayer?
A. Aurangzeb will face the north
B. Aurangzeb will face the south
C. Aurangzeb will face the north west
D. Aurangzeb does not pray at all

28/40. Cricket being India`s national sport is much loved by the countrymen. The cricket season is almost like a
festive season. If a game is played in any of the Indian cities, the stadium is full and tickets are even sold
in black. Most boy babies are presented with a plastic bat and ball in a bid to start them early. Parents
encourage their sons to attend summer cricket camps and spend huge amounts of money on cricket gear
and coaching. Which of the following statements can be inferred from the above passage?
A. India will produce many more Dravid`s and Tendulkar`s in future
B. Cricket is a religion in India.
C. Cricket reflects the universal gender bias as it is a man`s game.
D. Cricket encourages corruption in our country.
Directions: for each question in this section, select the best of the answer choices given.
27. The Mughal Emporer Aurangzeb observed certain rules in his daily prayers. He had to Mecca like all
Musilms do. During his morning prayer he was that the sun should not be in front of him or behind him.
The sun should not be the north, east, or west of him.
Which side will Aurangazeb face when doing his morning prayer?
A
Aurangazeb will face the north.
B
Aurangzeb will face the south.
C
Aurangzeb will face the north west.
D
Aurangzeb does not pray at all.
28/40. Cricket being India`s national sport is much loved by the countrymen. The cricket season is
almost like a festive season. If a game is played in any of the Indian cities, the stadium is full
and tickets are even sold in black. Most boy babies are presented with a plastic bat and ball in a
bid to start them early. Parents encourage their sons to attend summer cricket camps and spend
huge amounts of money on cricket gear and coaching.
Which of the following statements can be inferred from the passage ?
29/40. Directions: for each question in this section, select the best of the answer choices given Christopher
Marlowe was Shakespeare`s contemporary and wrote just as well though not too well known. One of his
most famous plays is the Tragical history of Dr. Faustus that was inspired by Goethe`s Faustbach. The
Tragical History of Dr. Faustus is a reader`s delight with exquisite phrases and expression. However,
comic relief in the play is ribald and vulgar and out of tune with the rest of the play. This type of comic
relief is uncharacteristic of Marlowe and is found nowhere in his other works like The Jew of Malta or
Tamburlaine.
Which of the following statements is a logical conclusion to the above?
A. The out-of-tune comic relief is now thought to be external interpolations.
B. His rival printed the text with the additions.
C. He unused such cheap comic relief to popularize his works.
D. Marlowe was afflicted with a psychosomatic disease when he wrote Dr.Faustus.
30/40. Postmodernism as a theoretical approach questions the structure of knowledge. Postmodernist critics have
even questioned the validity of the grand structure of science. The physicist, Alan Sokal termed
postmodernism as `fashionable nonsense` and dashed off a ` fashionable` article to the journal, Social
Text. Much to his amusement and satisfaction, it was published.
Which of the following statements is a logical conclusion to the above?
A. Alan Sokal is a cheat and a hoax.
B. The academic world was surprised that a physicist could write on humanist fields.
C. Alan Sokal was vindicated in his stand when he exposed the `uselessness` of Postmodernism.
D. Alan Sokal quit the field of physics and switched over to Postmodernism.
31/40. Booms and slumps are part of economic cycles just as inflation and deflation. The US has experienced an
economic depression in the past and is now experiencing an economic boom. There is a huge demand
for software and software projects are off shored to India thus triggering off a huge demand for software
specialists in India.
Which of the following statements is a false assumption?
A. Software specialists command high salaries in India.
B. The next stage in the drama of the US economy is a slump.
C. India is a favoured off shoring destination because of reasonable labour costs.
D. India has a huge pool of software talent.

32/40. The dodo lived on the island of Mauritius before it became extinct. The arrival of humans was the prime
cause of its extinction. Humans brought with them pets like cats and dogs that chased and killed dodos.
Besides, dodos were also favoured by humans as a tasty source of meat. The dodo did not know how to
defend itself as it could not even climb trees.
Which of the following statements can be deduced from the above passage?
A. `as dumb as a dodo` is right.
B. There were no trees on the island of Mauritius.
C. Humans are even crueller than animals.
D. The dodo was a flightless bird.
33/40. The International schools, initially set up to cater to children of foreign nationals employed in the city and
the children of NRI`s to imbibe Indian culture, is increasing seeing admissions from children of local
families too.
A. Is increasingly seeing admissions of children for local families too.
B. Are increasingly seeing admissions from children of local families too.
C. Are also increasingly seeing admissions from children of local families too.
D. Is seeing increasing admissions from children of local families too.
34/40. Encouraging women`s education is not about creating quotas for them in educational institutions,
increasing the number of seats, or offering financial assistance, though all that would help; it is about
the change in the attitudes and mindsets of men.
A. It is to change the attitudes and mindsets of men.
B. It is about changing the attitudes and mindsets of men.
C. It is for changing the attitudes and mindsets of men.
D.
It
is
a
change
about
the
attitudes
and
mindsets
for
men.
35/40. We could capitalize on unprecedented international attention, move quickly to rectify ills such as
moribund infrastructure and excessive red-tapism and had gone all out to make investors feel awesome.
A. Went all out to take investors feel
B. Will go all out to make investors feel
C. Go all out to make investors feel
D. May have gone all out to make investors feel
36/40. Ever since the first Indian went out to the far-flung corners of the world ,they have sought to be loyal
citizens of their adopted homelands.
A. They have been seeking to be loyal citizens for adopted homeland.
B. They have tried to become loyal citizens in their adopted homeland.
C. He has sought to be a loyal citizen of his adopted homeland.
D. He has sought out to be loyal citizens of their adopted homeland.
37/40. Every year we wait impatiently for the rain to begin, keep looking up expectant at the sky , discuss when
the monsoon will actually arrive and yet once the rains comes, it is only a matter of time before we
started cursing it.
A. It would be only a matter of time before we start cursing it.
B. It is only a matter of time before we will start cursing it
C. It was only a matter of time before we had started cursing it,
D. It is only a matter of time before we start cursing it.
38/40. India , clout emerges more from political power as bussiness power , something that`s noy understand in
the west.
A. More from political power than of business power
B. More from political power than business power
C. More from power as in business power
D. More from political power then after from business power
39/40. When new identities shaped by economic forces and urbanization are formed,
and ethnic bonding might begin to recede.
A.Old forms of linguistic and ethnic bonding may begin to recede.
B. Old forms of linguistic and ethnic bonding may began to recede.
C. Old forms of linguistic and ethnic bonding may have begun to recede.
D. There may be old forms of linguistic and ethnic bonding beginning to recede.

old forms of linguistic

40/40. As we drift away from our agrarian roots and as our ability to control our immediate and increases, our
relationship with the rain too has begun to change.

A. Our relationship with the rain too will begin to change


B. Our relationship with the rain too will have begun to change
C. Our relationship with the rain too will begin to change
D. Our relationship to the rain too is beginning to change
E. Our relationship with the rain too begins to change
Directions: For each question in this section, select the best of the answer choices given.
1/40. Striking results have been obtained from investigations of the general standard of health of
different income groups. Public school boys, who on the whole come from better fed
families, are taller, heavier and stronger than secondary school boys. Moreover, infantile
death rate among children of labourers was four times as great as it was among the wellto-do children.
It can be inferred from the passage that,
A.
B.
C.
D.

Children from high income groups overate.


Even if poorly fed children are given better diet they cannot improve.
Although nutrition is only one factor of difference, it can hardly be without significance.
Public school boys are given more training and physical activities to become taller and stronger.

2/40. Ironically enough, anorexia appears to exist only in the section of society affluent enough to feed people
well. Where survival is a struggle, anorexia is rare. An explanation of that theory is that girls from
affluent homes grow up with more rationalisation powers, a greater concern about their bodies and
general appearance and therefore, have to fit into the mould that is expected of them.
It can be assumed that
A.
Most young girls have a tendency to believe that beauty lies in the way other people want you to
look.
B.
Anorexia is a serious problem that is calling for attention.
C.
Eating disorder is prevalent mostly among women than it is among men.
D.
Anorexia is still a subject of research.
3/40. When Olympics were first started in Greece, it was an occasion for harmony, understanding,
relaxation and promoting peace. The revival of the Olympiad ideally fitted in the gentlemans concept of
gallantry in games in the latter part of the 19th century. Slowly but steadily, politics started penetrating
into the field of sports.
Which of the following is an evidence of the claim made by the argument ?
A.
Race, religion and creed are important determiners for participation in Olympics .
B.
Before the 19th century the main purpose of the Olympics was competition and business.
C.
India remained supreme in hockey as long as there was no politics mixed with games.
D.
China is ahead of India in sports as well.
4/40. From October 1, it will be mandatory for all refineries to blend at least 5% of ethanol with petrol. The
government is enthusiastically pursuing the fuel called Jetrol, which increases efficiency. Therefore, the
problem of fuel prices is also met. The government is very happy about this new finding, which it will
introduce in another weeks time.
Which of the following statements weakens the argument?
A.
The fuel thus produced emits pollutants which are three times more dangerous than that emitted
by the existing fuel.
B.
The fuel, jetrol is highly affordable by almost all segments of the population.
C.
The price of spirit is also likely to increase considerably.
D.
Many industries have now begun to manufacture ethanol.
5/40. The government should ensure that the prices of drugs do not go up. All taxes and duties on drugs should
be withdrawn as it taxes the suffering humanity. Drugs, undoubtedly, are an important medical resource.
It can be inferred from the passage that
A.
It is important to weed out unnecessary drugs
B.
Drugs have the potential for improving the efficiency and effectiveness of medicine
C.
Doctors should be aware of the new drug delivery systems
D.
Drug therapy must be given more importance.
6/40. Without new-generation computer chips made in japan, the US department of defence cannot
guarantee precision of its nuclear weapons. If japan told Washington that it would no longer sell
computer chips to the united states, the pentagon would be totally helpless and in dangerous
position.
Which of the following statements strengthens the argument?
A.
The accuracy of the missiles depends on the quality of the computers.

B.
C.

The Americans can do a little better than depend on the intelligence of other countries.
Japan can completely upset the American military balance if it sells its computer chips to the
Soviet Union instead of to the US.
D.
The Americans have been wise in calling off the arms race.
Directions: Fill in the blank with the correct option that fits in grammatically and logically.
7/40. He had ___________ his face grow larger than they should.
A.
A unique disease that made the bones from.
B.
A rare disease that made the bones of.
C.
An unnatural disease that will make the bones of.
D.
A special disease that made the bones in.
8/40. They _________ TV ever since they came home.
A.
Watched
B.
Have watched.
C.
Have been watching.
D.
Should watch.
9/40. The long, lone hours on trial gave Doug a chance _____________.
A.
To get himself to know better.
B.
For getting to know himself better.
C.
To get to know himself better.
D.
In getting better to know himself.
10/40. That is a moving story ___________.
A.
That had survived the centuries.
B.
That has survived the centuries.
C.
Which is survived by the centuries.
D.
That is a survival in the centuries.
11/40. The girl ________ a defect in her nervous system.
A.
Was unable in feeling pain because of.
B.
Was disabled to feel pain in.
C.
Was incapacitated to feel pain because of.
D.
Was unable to feel pain because of.
12/40. As soon as the plane ____________ to a hospital.
A.
Landed, he was rushed.
B.
Was landed, he was rushed.
C.
Was grounded, he was rushed.
D.
Could be landed, he went rushing.
13/40. You try to _________ when you want to control your laughter.
A.
Put a brave face.
B.
Make a face.
C.
Keep your hair on.
D.
Keep a straight face.
14/40. When it comes to English grammar, she certainly __________.
A.
Gets the wrong end of the stick.
B.
Rings a bell.
C.
Knows her stuff.
D.
Carries the can.
Questions: Identify the correct sentence.
15/40.
1.
Men in my office love to take longer coffee breaks.
2.
The men in my office love taking long coffee breaks.
3.
Men at my office take long coffee breaks and they love it.
4.
Taking long breaks for coffee is what men in my love to do.
A.
1
B. 2
C. 3
D. 4
16/40.
1.
2.
3.
4.
A.

This years college trip to Simla was the most enjoyable vacation I have ever had.
This years college trip to Simla was a more enjoyable vacation I had.
The most enjoyable college trip I ever have was the Shimla trip of this year.
The most enjoyed this years college trip in Simla to the utmost.
1
B. 2
C. 3
D. 4

1.

If Ram doesnt pay the mortgage to his house on time, he will be out in the street.

17/40.

2.
3.
4.
A.

Ram will be out in the street if he does not pay mortgages of his house in time.
If Ram does not pay mortgage to his house he will be out of the streets on time.
If Ram does not pay the mortgage on his house on time, he will be out of the streets on time.
1
B. 2
C. 3
D. 4

1.
2.
3.
4.
A.

My laptop was stolen at the airport yesterday while I was waiting.


They had stolen my laptop yesterday at the airport while I was waiting.
I was waiting at the airport yesterday when they stole my laptop.
While I was waiting at the airport yesterday, my laptop was stolen.
1
B. 2
C. 3
D. 4

1.

A.

General intelligence is essential for outstanding achievement because it involves ones natural
ability to comprehend difficult concepts quicker and to analyse them clearly.
General intelligence is essential in outstanding achievement because it involves the natural
ability of one in comprehending difficult concepts quicker and to analyse them clearly.
General intelligence is essential for outstanding achievement because it involves ones natural
ability to comprehend difficult concepts quicker and analysing them with clearness.
General intelligence is essential for outstanding achievement because it naturally involves ones
ability to comprehend difficult concepts quicker and an analysis of them clearly.
1
B. 2
C. 3
D. 4

1.
2.
3.
4.
A.

I enjoyed a visit to the museum; it was far interesting than I expected.


My visit to the museum by far interesting so I enjoyed it.
I enjoyed the visit to the museum; it was far more interesting than I had expected.
I enjoyed our visit to the museum as it was far more interesting than we expected.
1
B. 2
C. 3
D. 4

1.

To prepare himself, he subjected himself to two weeks of intensive training on the Indonesian
island of Java, climbing the volcano near Bandung 25 times, spend nights in the open and to test
to see how long he could hold out without food.
To prepare himself, he subjected himself to two weeks intensive training on the Indonesian
island of Java, climbing the volcano near Bandung 25 times, spending nights in the open and to
test to see how long he could hold out without food.
In preparation for himself, he subjected himself to two weeks of intensive training on the
Indonesian island of Java, climbing the volcano near Bandung 25 times, spending nights in the
open and testing to see how long he could hold out without food.
To prepare himself, he subjected himself to two weeks of intensive training on the Indonesian
island of Java, climbing the volcano near Bandung 25 times, spending nights in the open and
testing to see how long he could hold out without food.
1
B. 2
C. 3
D. 4

18/40

19/40.
2.
3.
4.
20/40

21/40

2.
3.
4.
A.
22/40.

1.
The hostess found out that there was no more drink in Arvinds glass.
2.
The hostess noticed that there was no more drink in Arvinds glass.
3.
The hostess noticed that Arvins glass could contain no more drink.
4.
The hostess identified that Arvinds had no more to drink in his glass.
A.
1
B. 2
C. 3
D. 4
Directions: In each of the following sentences, some part of the sentence or the entire sentence is underlined.
Beneath each sentence you will find four ways of phrasing the underlined part. This is a test of
correctness and effectiveness of expression. In choosing answers, follow the requirements of standard
written English that is, pay attention to grammar, choice of words, and sentence construction. Choose
the answer that expresses most effectively what is presented in the original sentence; this answer should
be clear and extract, without awkwardness, ambiguity or redundance
23/40. Further studies are needed to better circumstantiate these conditions and for providing safe rules for the
use of these increasingly more widespread device.
A.
And for provision of safe rules to use these increasingly more.
B.
And to provide safe rules to using of these increasingly more.
C.
And also providing safe rules for using of this increasingly more.
D.
And to provide safe rules for the use of this increasingly more.

24/40. Globalization too uses economic, social and political relationships as well for creating new boundaries
that are not dependent on geographical proximity.
A.
Social and political relationships as well for creating.
B.
Social and political relationships to create.
C.
Social and political relationships that create.
D.
Relationships social and political as well to create.
25/40. The purpose of the Vedas should ensure the true welfare of all beings, worldly as well as spiritual.
A.
Was to ensure the true welfare of all beings, worldly as well as spiritual.
B.
Is to ensure the welfare of all beings that are both worldly and spiritual.
C.
Was to insure both worldly and spiritual welfare of all beings.
D.
Was to make sure that the true welfare of all beings is worldly as well as spiritual.
26/40. Sceptics emphasize the fact that many of the conditions said to be related to allergy fluctuate in severity
and also has a significant psychological component.
A.
And will have a significant psychological component.
B.
And have a significant psychological component.
C.
And had a significant psychological component.
D.
And also must be having a significant psychological component.
27/40. Dr. Abdul Kalamsboog ignited Minds is simplistic and thought provoking.
A.
Is simplified.
B.
Was simplistic.
C.
Has been simple
D.
Is simple.
28/40. Lying on the bed, the movie was watched by her.
A.
The movie was watched by her.
B.
The movie was being watched.
C.
The movie is watched.
D.
She watched the movie.
29/40. Fear of any particular enemy is definitely to be an instinctive quality, though it is strengthened by
experience, and by the sight of fear of the same enemy in other animals.
A.
Is certain to be an instinctive quality.
B.
Is of a certainly instinctive quality,
C.
Is certainly an instinctive quality,
D.
Is surely a certain instinctive quality
30/40. The Dutch Health Council analysed serval studies and found none of the evidence that radiation from
mobile phones were harmful.
A.
None of the evidence that radiation from mobile phones was harmful.
B.
Is certain to be an instinctive quality,
Directions: Read the following passage below and answer the questions that follow on the basis of what is stated
/ implied in that passage.
DJ VU
A.
In the summer of 1856, Nathaniel Hawthorne visited a decaying English manor house known as
Stanton Harcourt, not far from Oxford. He was struck by the vast kitchen, which occupied the
bottom of a 70 foot tower. Here, no doubt, they were accustomed to roast oxen whole, with
as little fuss and ado as a modern cook would roast a fowl, He wrote in an 1863 travelogue,
Our Old home.
B.
Hawthorne wrote that as he stood in that kitchen, he was seized by an uncanny feeling: I was
haunted and perplexed by an idea that somewhere or other I had seen just this strange spectacle
before. The height, the blackness, the dismal void, before my eyes, seemed as familiar as the
decorous neatness of my grandmothers kitchen. He was certain that he had never actually seen
this room or anything like it. And yet for a moment he was caught in what he described as that
odd state of mind wherein we fitfully and teasingly remember some previous scene or incident,
of which the one now passing appears to be but the echo and reduplication.
C.
When Hawthorne wrote that passage there was no common term for such an experience. But by
the end of the 19th century, after discarding false recognition. Par amnesia, and promnesia,
scholars had settled on or a French candidate: dj vu, or already seen.
D.
The fleeting melancholy and euphoria associated with dj vu have attracted the interest of
poets, novelists, and occultists of many stripes St. Augustine, Sir Walter Scott, Dickens, and
Tolstoy all wrote detailed accounts of such experiences.
E.
Most academic psychologists, however, have ignored the topic since around 1890, when there
was a brief flurry of interest. The phenomenon seems at once too rare and too ephemeral to
capture in a laboratory. And even if it were as common as sneezing, dj vu would still be
difficult to study because it produces no measurable external behaviours. Researchers must trust

their subjects personal descriptions of what is going on inside their minds, and few people are
as eloquent as Hawthorne. Psychology has generally filled dj vu away in a drawer marked
Interesting bust insoluble
F.
During the past two decades, however, a few hardy souls have reopened the scientific study of
dj vu. They hope to nail down persuasive explanation of the phenomenon, as well as shed
light on some fundamental elements of memory and cognition. In the new book The Dj vu
Experience: Essays in cognitive Psychology (Psychology Press), Alan S. Brown, a professor of
psychology at Southern Methodist University. Surveys the fledgling sub field. What we can try
to do is zero in on it from a variety of different angles, he says. It wont be something like,
Boom! The explanation is there. But we can get gradual clarity through some hard work.
31/40. Uncanny in the context means,
A. Audacious
B. Terrible
C. Irrational
D. Strange.
32/40. Ephemeral in paragraph E means,
A.
Short - lived
B.
Insignificant
C.
Irrelevant
D.
Unrealistic
33/40. The passage mainly,
A.
Evaluates the phenomenon of dj vu.
B.
Criticizes the psychologists reluctance to research into the phenomenon.
C.
Elaborates on the need for research into dj vu.
D.
Discusses the reason why the phenomenon is difficult to research.
34/40. It can be inferred from the passage that dj vu .
A.
Is a commonplace occurrence.
B.
Is a manifestation of the subconscious?
C.
Is an inexplicable and rare event.
D.
Cannot be researched scientifically.
35/40. Which of the following statements is true according to the passage?
A.
Dj vu as a phenomenon started happening in the 19th Century.
B.
Researchers have been reluctant to show interest in the phenomenon.
C.
It is not always easy to describe the thought in ones mind.
D.
Dj vu is another word for par amnesia.
E.

Passage 1:
General Agreement on Tariffs and Trade (typically abbreviated GATT) functioned as the precursor to the World
Trade Organization. GATT was created by the Bretton Woods meetings to take place in Bretton Woods,
New Hampshire, in 1944, which set forth a plan for economic recovery after World War 2 by
encouraging a reduction in tariffs and other international trade barriers.
On January 1, 1948 the agreement was signed by 23 countries: Australia, Belgium, Brazil, Canada, Ceylon,
Chile, China, Cuba, the Czechoslovak Republic, France, India, Lebanon, Luxembourg, Netherlands,
New Zealand, Norway, Pakistan, Southern Rhodesia, Syria, South Africa, the United Kingdom, and the
United states.
This first version, developed in 1947 during the United Nations Conference on Trade and Employment in
Havana, Cuba is referred to as GATT 1947. In 1994, GATT was updated (GATT 1994) to include
new obligations upon its signatories. One of the most significant changes was the creation of the World
Trade Organization (WTO). The 75 existing GATT members and the European Communities became
the founding members of the WTO on January 1, 1995. The other 52 GATT members have joined and
28 are currently negotiating membership. Of the original GATT members, only the SFR Yugoslavia has
not re-joined. Since FR Yugoslavia, (renamed to Serbia and Montenegro and with membership
negotiations later split into two), is not recognized as a direct SFRY successor state; therefore, its
application is considered a new (non-GATT) one. The contracting parties who founded the WTO ended
official agreement of the GATT 1947 terms on December 31, 1995.
The GATT, as an international agreement, is similar to treaty; under United States law it is classified as a
congressional executive agreement. The agreement is based on the unconditional most favoured nation
principle. This means that the conditions applied to the most favoured trading nation (i.e. the one with
the least restrictions) apply to all trading nations.
While the United States has always participated in international trade, it did not take a leading role in global
trade policy making until the Great Depression. One reason for this is that under the US Constitution the
congress has responsibility for promoting and regulation commerce, while the executive branch has
responsibility for foreign policy. Therefore, trade policy became a tug-of-war between congress and the
Executive branch in deciding the mix of trade promotion and protectionism. However, the United States
began to experiment with the reciprocal trade agreements act of 1934 an in the hopes of expanding
employment, congress agreed to permit the executive branch to negotiate bilateral trade agreements.
During the 1930s, the amount of bilateral negotiation under this act was fairly limited, and consequently did
little to expand global. Near the end of the Second World War US policy makers began to experiment on
a broader level. In the 1940s, working with the British government, the United States developed two
innovations to expand and govern trade among nations; the general agreement on tariffs and trade
(GATT) and the International Trade Organization (ITO). GATT was a temporary multilateral agreement
designed to provide a framework of rules and a forum to negotiate trade barrier reductions among
nations. Based on the reciprocal trade agreements act it allowed the executive branch negotiating power
over trade agreements with temporary authority from congress. At the time it functioned as a
provisional, but promising trade system.

Whereas GATT was a set of rules agreed upon by nations, the WTO was an institutional body. The WTO
expanded its scope from traded goods to trade within the service sector and intellectual property rights.
Although it was designed to sever multilateral agreements, during several Rounds of GATT
negotiations (particularly the Tokyo Round) plurilateral agreements created selective trading and caused
fragmentation among members. WTO arrangements are generally a multilateral agreement settlement
mechanism of GATT.
36/40. The United States did not take a leading role in global trade policy making until the great depression
because:
A.
The US constitution did not allow it.
B.
The congress fell behind in taking decisions.
C.
Bilateral trade agreements were not allowed.
D.
Of internal politics between the congress and the executive branch.
37/40. What was the significance of GATT 1994?
A.
The signatories had more obligations.
B.
The world trade organisation was created.
C.
It had expanded to include 75 members.
D.
Congo was the first African to join GATT.
38/40. There was dissension among members of the WTO.
A.
Because of multilateral agreements put forth by GATT.
B.
Because of WTO including the service sector under its purity.
C.
Plurilateral agreements led to selective trading.
D.
The members did not agree upon the intellectual property rights.
39/40. What was GATTs primary agenda?
A.
To set up the WTO.
B.
To set up an international cartel.
C.
To formulate a plan for economic recovery after World War 2.
D.
To smoothen out trade between the developed and developing countries.
40/40. According to the passage.
A.
Twenty one new non-GATT members joined after the founding of the WTO.
B.
The GATT is a congressional-executive agreement.
C.
Twenty eight countries are negotiating for WTO membership.
D.
All of the above.
DIRECTIONS: Read the following passage below and answer the questions that follow on the basis of what is
stated / implied in that passage
Shakespeare is known as one of the worlds greatest playwrights. He has written tragedies, histories, and
sonnets. But one of Shakespeares greatest talents was writing comedies. He used many techniques
when writing a comedy and some of these seem to be consistent throughout his comedies.
One of the first technique that should be discussed is the subject matter of Shakespeares comedies. Shakespeare
always uses love and marriage as the content for his comedies. This can be seen in the comedies much
ado about nothing, As You Like it, A Midsummer Nights Dream, and The Taming of the Shrew, where
the characters fall in love and get married. Another technique that Shakespeare incorporates into his
comedies is the use of the lower class for comedy. Shakespeare tends to poke fun at the lower class and
make them into fools in his comedies. An example of this is Much Ado about nothing, where
Shakespeare has the constable Dogberry and his foolish assistant verges run acting like they are riding
horses. Another example of Shakespeares use of the lower class for comedy is in The Taming of The
Shrew. In that comedy Shakespeare makes Petruchios servants bumbling and incompetent.
The use of eavesdropping is another very important device in Shakespeares comedies. This technique plays a
major role in Much Ado About Nothing where it is used to get Benedick and Beatrice together. Also it is
used considerably in As You Like it.
The last technique is the one everyone knows and loves the happy ending. Shakespeare consistently has a
happy ending in his comedies. These happy endings usually involve the lovers finally getting together
and getting married after they have solved the problem that had been keeping them apart. This can be
seen in Much Ado About Nothing where the situation between Claudio and Hero was cleared up and
they were able to marry. Also Bendick and Beatrice were able to marry with the help of their friends
who showed them how they truly felt about each other. Similar situations occur in Shakespeares other
plays, the overall result being a touching and enjoyable ending to the comedy.
As it can be seen Shakespeare was an excellent comedy writer. He used many techniques in his comedies and
often these techniques would be reused in other comedies. With such skill in writing these plays and the
intuitive use of these techniques it is no wonder why we treasure Shakespeares comedies.
1/40.
An example of making fads of the lower class is seen in

A.
B.
C.
D.

Midsummer Nights Dream


As You Like it
A comedy of errors
Much Ado About Nothing

2/40
2/40
What is the overall result of the most loved technique?
a. Happy ending
b. Comedy
c. Touching and enjoyable ending
d. Love stories
3/40
Which word is a synonym for Eavesdrop?
a. Fall
b. Drip
c. Spy
d. abandon
4/40
Shakespeares most consistent ending involves
a. comedy
b. lovers getting together and marrying
c. making fun of the lower fad
d. sonnets
5/40
What can you infer from this passage?
a. Shakespeare is known as one of the worlds greatest playwrights
b. Shakespeare consistently wrote happy endings in his comedies.
c. Shakespeare incorporates into his comedies is the use of the lower class for comedy.
d. Shakespeare seemed to be consistent throughout his comedies
Directions: in each of the following sentences, some part of the sentences of the entire sentence is
underlined. Beneath each sentence you will find four ways of phrasing the underlined part. This is a test
of correctness and effectiveness of expression. In choosing answers, follow the requirements of standard
written English that is, pay attention to grammar, choice of words, and should be clear and exact without
awkwardness, ambiguity or redundancy.
6/40.
Early inthis century, afterbecoming the first Black to receive a Ph.D. at Harvard, a famous essay arguing the fate
of the Negro race was written by Du Bois.
A.
B.
C.
D.

a famous essay arguing the fate of the Negro race was being written by Du Bois.
a famous essay arguing the fate of the Negro race had been written by Du Bois.
a famous essay arguing the fate of the Negro race having been written by Du Bois.
Du Bois wrote a famous essay arguing the fate of the Negro race.

7/40.
The process must being with clear communication and education on why lean manufacturing is beneficial and
removes the employees fear that they will lose their jobs.
A.
B.
C.
D.

Remove the employees fear


For removal of fear in the employees
will remove the employees fear
should have removed the employees fear

8/40
From going to college and doing a part time job, Jamie also finds time for sports and fitness.
A.
B.
C.
D.

Apart from going to college and doing a part time job


Though going to college and a part time job
In spite of doing college and a part time job
Despite a job and college

9/40.
The reward of the spiritual experience is that it enables each individual to grow and wholly to realize the
tremendous potential unique to that person.

A. totally realizes the tremendous potential unique to


B. fully realizing the tremendous potential unique to
C. fully realize the tremendous potential unique to
D. fully realize the tremendous potential unique in
10/40.
A human being functions not only through the physical body but through the use of the intellectual and the will
A. but through the usage of the intellectual skills
B. but by the use of the intellect
C. but also through the use of the intellect
D. but because of the intellectual use of
11/40
The knowledge that the public possess on any important issue is derived from vast and powerful organisations
the press, radio, and above all, the television.
A. That the public has possession of
B. That the public possess on
C. That the public are known possess in
D. That the public has possessed on
12/40
Stress illness comprises of perhaps as much as 75 percent of all of todays illness
A. Comprises perhaps so much as
B. Comprises of perhaps as many as
C. Comprises maybe as many of
D. Comprise perhaps as much as
13/40
The ability to distinguish between morally good and evil acts it critical towards the formation in character.
A. Is critical to the formation of character
B. Are critical towards character forming
C. Is so critical in formation in character
D. Is absolute towards the formation of character
Directions: Read the following passage below and answer the questions that follow on the basis of what is stated
/ implied in that passage.
Animal communication is any behaviour on the part of one animal that has an effect on the current or future
behaviour of another animal. The study of animal communication, called zoomsemiotics
(distinguishable from anthroposemiotics, the study of human communication) has played an important
part in the development of ethology, sociobiology, and the study of animal cognition. Animal
communication, and indeed the understanding of the animal world in general, is a rapidly growing field,
and even in the 21st century so far, many prior understandings related to diverse fields such as personal
symbolic name use, animal emotions, animal culture and learning, and even sexual conduct, long
thought to be well understood, have been revolutionized.
The sender and receiver of a communication may be of the same species or of different species. The majority of
animal communication is intraspecific (between two or more individuals of the same species). However,
there are some important instances of interspecific communication. Also, the possibility of interspecific
communication. Also, the possibility of interspecific communication, and the form it takes, is an
important test of some theoretical models of animal communication.
If a prey animal moves or makes a noise in such a way that a predator can detect and capture it, it fits the
definition of communication given above. Nonetheless, we do not feel comfortable talking about it as
communication. Our discomfort suggests that we should modify the definition of communication in
some way; either by saying that communication should generally be to the adaptive advantage of the
communicator, or by saying that it involves something more than the inevitable consequence of the
animal going about its ordinary life.
There are however some actions of prey species that are clear communications to actual or potential predators. A
good example is warning colouration: species such as wasps that are capable of harming potential
predators are often brightly coloured, and this modifies the behaviour of the predator, which either
instinctively or as the result of experience will avoid attacking such an animal. Some forms of mimicry
fall in the same category: for example hoverflies are coloured in the same way wasps, and although they
are unable to sting, the strong avoidance of wasps by predators gives the hoverfly some protection.
There are also behavioural change and warning colouration will be combined, as in certain species of
amphibians which have a brightly coloured belly, but on which the rest of their body is coloured to blend
in with their surroundings. When confronted with a potential threat, they show their belly, indicating that
hey are poisonous in some way. A more controversial example of prey to predator communication is
stotting, a highly noticeable from of running shown by some antelopes such as thomsons gazelle in the

presence of a predator; it has been argued that this demonstrates to the predator that the particular prey
individual is fit and healthy and therefore not worth pursuing.
Some predators communicate to pray in ways that change their behaviour and make them easier catch, in effect
deceiving them. A well-known example is the angler fish, which has a fleshy growth protruding from its
forehead and dangling in front of its jaws; smaller fish try to take the lure, and in doing are perfectly
placed for the angler fish to eat them. Interspecies communication also occurs in various kinds of
mutualism and symbiosis. For example, in the cleaner fish/ grouper system, groupers signal their
availability for cleaning by adopting a particular posture at cleaning station.
Various ways in which humans interpet the behaviour of domestic animals, or give commands to them, fit the
definition of interspecific communication. Depending on the context, they might be considered to be
predator to prey communication , or to reflect forms of commensalism. The recent experiments on
animal language are perhaps the most sophisticated attempt yet to establish human/animal
communication, though their relation to natural animal communication is uncertain.
14/40. According to the passage
A. The sender and receiver of a communication have to be of the same species
B. A human communicating to animals is an example of interseptic communication
C. Only domestic animals respond to human commands
D. Animals cannot follow commands given by human beings
15/40. A proper definition of animal communication should include:
A. Signalling between two or more individuals of the same species
B. The involvement of something morethan the inveitable result of an animal going about is daily life
C. The detection of prey animal by predator
D. Communication that has an effect on the current or future behaviour of another animal
16/40The statement There are also behavioural change and warning colouration will be combined, as in certain
species of amphibians which have a brightly coloured belly, but on which the rest of their body is
coloured to blend in with their surroundings can be described as
a. Camouflage
b. Mime
c. Cover
d. Defence
16/40
Which of the following statement is not true.
a. The sender and receiver of a communication may be of the same species or of different species.
b. The possibility of interspecific communication, and the form it takes, is an important test of some theoretical
models of animal communication.
c. They change their color to find food.
d. The majority of animal communication is intraspecific
18/40
Explain stotting.
a. Animals showing off their fighting capabilities
b. Animals showing off their eating capabilities
c. Animals showing off their strength and speed
d. Animals pretend to be dead.

QUESTIONS: Identify the correct sentence


19/40.
1. Men in office love to take longer coffee breaks
2. The men in my office love taking long coffee breaks
3. Men at my office take long coffee breaks and they love it
4. Taking long breaks for coffee is what men in my office love to do
20/40.
1. This years college trip to simla was the most enjoyable vacation I have ever had
2. This years college trip to simla was more enjoyable vacation I had
3. The most enjoyable college trip I ever have was the simla trip of this year
4. I have enjoyed this years college trip in Simla to the utmost
21/40
a. Sometimes it is difficult to live a honest life.
b. Sometimes it is difficult to live an honest life.
c. Difficult it is sometimes to live honest life
d. Sometime it is difficult to live an honest life

DIRECTIONS: For each question in this section, select the best of the answer choices given

A.
B.
C.
D.

Ram Rakhi knows the route to the stadium


Ram Raka doesnt know german
Ram rakhi is trying to put down Ram Raka
The Two of them are on a virtual tour

A. Something has to be done about the noise pollution in Bangalore


B. The BMP is doing nothing to widen the roads
C. It is high time that officials started strict pollution checks, ensured emission control, and cancelled licenses of
those who did not follow traffic rules
D. The government should ban diesel autos
29/40
1.
The renaissance in Britain's car industry has been a beacon of hope in an economy that has been
broadly flat for the best part of two years. It can help rebalance the UK economy by increasing
our manufacturing output and our exports. It can also create high quality jobs. But we shouldn't
be under too many illusions: even if manufacturing companies like Jaguar can help increase UK
exports, many economists doubt they can be a major source of new jobs over the long term.
Which of the following best explains the reason for the doubt expressed by the economists?

A.
The
car
industry
does
not
create
low-quality
jobs.
B. Even though Jaguars exports grew more than 20% in the last year, its number of employees
grew
only
by
5%.
C. The car industry is a capital intensive industry and frequently goes through cycles of growth
and
recession.
D. To succeed in the global manufacturing industry, a company has to be extraordinarily
efficient, which means using the same workforce for more production.
E. US car companies are financially well off than their British peers and can cut down their
prices to gain market share.
30/40
The decline of manufacturing in a region sets off a chain reaction. Once manufacturing is outsourced,
process-engineering expertise cant be maintained, since it depends on daily interactions with
manufacturing. Without process-engineering capabilities, companies find it increasingly difficult to
conduct advanced research on next-generation process technologies. Without the ability to develop
such new processes, they find they can no longer develop new products.
Which

of

the

following

is

best

supported

by

the

passage?

A. In the long term, an economy with a deteriorating infrastructure for advanced process
engineering and with declining manufacturing activity will likely lose its ability to innovate.
B. To develop new products, a company must never cut down on its manufacturing.
C. An increase in the manufacturing activity in a region leads to increase in innovation within the
region
D. To develop new products, a company needs to frequently change its processes.
E. An economy that has growing manufacturing capabilities will churn out more new products than
other economies.
31/40
Whether you can accomplish a specific goal or meet a specific deadline depends first on how much time
you need to get the job done. What should you do when the demands of the job exceed the time
you have available? The best approach is to divide the project into smaller pieces. Different goals
will have to be divided in different ways, but one seemingly unrealistic goal can often be
accomplished by working on several smaller, more reasonable goals.
The main idea of the passage is that
a.
jobs often remain only partially completed because of lack of time.
b.
the best way to complete projects is to make sure your goals are achievable.
c.
the best way to tackle a large project is to separate it into smaller parts.
d.
the best approach to a demanding job is to delegate responsibility.
32/40
Health clubs have undergone a major transformation that can be described in three words: mind, body,
and spirit. Loud, fast, heartt-humping aerobics has been replaced by the hushed tones of yoga and
the controlled movements of Pilates. The clubs are responding to the needs of their customers who
are increasingly looking for a retreat from their hectic lifestyles and a way to find a healthy
balance in their lives by nurturing their whole selves.
The main idea of the paragraph is that
a. exercise is less important now than it once was.
b. health clubs are much less popular now than they were ten years ago.
c. many health clubs will go out of business because of the decline in traditional exercise.
d. people's desire to nurture all aspects of themselves has contributed to big changes for health clubs.
Directions: Fill in the blank with the correct option that fits in grammatically and logically.
33/40. The doctor went home,____________he?
A. Has
B. Did
C. Doesnt
D. Didnt
34/40.
The number of people in the stadium was__________
A. Penty
B. Scarcity
C. Many
D. Unimaginable
35/40. Hema is my best friend, Ive known her __________my school days

A. On
B. For
C. Since
D. Till
36/40. It is not impossible to go without food __________ a week
A. Till
B. As long as
C. For
D. Since
37/40.
When a body enters the earths atmosphere, it travels _____________
A. Very rapid
B. In a rapid manner
C. With great speed
D. At great speed
38/40.
The furniture in the room______________
A. Is antique
B. Is antic
C. Are antic
D. Were antique
39/40.
Employers often require that candidates have not only a degree __________________
A. But two years experience
B. Also two years experience
C. But also two years experience
D. But two years experience
40/40.
Professional people expect _______________ when it is necessary to cancel an appointment
A. You call to them
B. You to call them
C. That you must call them
D. Your calling them
Directions: For each question in this section, select the best answer choices given.
1/40.Unless you are a maradona, you require at least 6 feet in height, enormous stamina to win in modern field
games like soccer. Indians lack both, which is why they are mediocre in sports.
Which of the following statements weakens the argument?
1.
The players on the Italian team that won the world cup are an average of 5 feet in height. 2.
2.
Enormous stamina is a prerequisite for the sport.
3.
Since India has no Maradona, India cannot win a soccer tournament.
4.
Height is the major crucial factor that determines the mediocrity level in any sport.
2/40. Like many other legislations, the anti-tobacco drive, is also broken many a time, most often in court
complexes. Since India is the second largest producer of tobacco. The country has this year the dubious
distinction of affecting more and more youth of cancer, stroke and heart ailment, compared to other
nations. This accounts for the increasing death rate of the youth in our country from 14% to 28%
Which of the following statements supports the argument?
1.
Smoking tobacco does not cause serious heart ailments that can be lethal.
2.
Survey shows that in 20 Indian states, the new trend of youth smoking tobacco has been the
cause of the increased death rate.
3.
Passive smoking is not as injurious as active smoking.
4.
Anti-tobacco drive is quite effective in some cities.
3/40. The windmill company works in much the same way that a real estate developer would. It starts out by
identifying a plot of land and takes on an average 3 years to get approvals and develop it. Having done
that, it woes people with investable funds to buy a windmill on a plot. Therefore, a person who has
worked for a realtor is qualified to work for a wind powered company. It can be inferred from the
argument that:
1.
Since wind power industry is growing. It will be able to generate more than 15 lakh units of
power each year.
2.
The wind power industry will see its success at a faster rate than a realtor.
3.
More people in the real estate business are moving to wind power industry in recent times.
4.
The conclusion has been arrived at hastily based on a faculty analogy.

4/40. Political pressure has pushed a world bank funded e governance initiative to the back burner. The states
public works department worked out a futuristic initiative Road information System (RIS) that maps
the condition of each road in the state, the traffic density and wether it needs to repair or maintenance.
The PWD generates information and keeps hoping that someday it will come to use. It has, therefore,
become a mere statistics.
It can be assumed that:
1.
Funds for the road improvement depends on how powerful the local politician is.
2.
Politics is usually blamed for roadblocks.
3.
The PWD has not been informed about the RIS.
4.
Climatic conditions are the major reasons for roadways still remaining backward.
5/40. The Orissa government has planned to introduce a dress-code for college principals. They will
have to wear coat, suit, shoe and tie. The move is aimed at making the principles sport a smart look and
endure campus discipline.
Which of the following about the dress code is a false assumption in the argument?
1.
It will improve the academic ambience.
2.
It will ensure discipline in the campus.
3.
It will upgrade the academic performance of the students.
4.
It will give the principle a more professional appearance and boost their image.
6/40. The oil and gas exploration major, Hindustan Oil Corporation has been directed by the petroleum ministry
to stick to its core competence of oil and gas exploration rather than diverting focus on retailing products
of petroleum products. A directive to this effect was issued owning to the heavy losses incurred by
Hindustan oil Corporation which attempted such a venture in the past.
Which of the following statements is most seriously undermines the directive of the petroleum ministry?
1. There are very few oil exploration companies when the demand for oil and gas is ever increasing.
2.Indane, another gas and oil exploration unit, ventured into retailing of petroleum products and has seen a
tremendous growth within a considerably short period.
3. Petroleum products are not many; so there is no scope for the industry.
4. Oil and gas explorations is not as lucrative a business as is the retailing business.
Questions: Identify the correct sentence.
7/40.
A.
The visibility in consumption styles in our cities certainly effects both the kind and intensity of
the consumption in rural areas.
B.
The visibility in consumption styles in our cities certainly affects both the kind and intensity of
the consumption in rural areas.
C.
The visibility in consumption styles in our cities certainly affected both the kind and intensity of
the consumption in rural areas.
D.
The visibility in consumption styles in our cities certainly should affects both the kind and
intensity of the consumption in rural areas.
8/40.
A.
B.
C.
D.

If I were an architect, I would construct a skyscraper.


If I was an architect, I would construct a skyscraper.
If I am an architect, I would construct a skyscraper.
If I had been an architect, I would construct a skyscraper.

A.

What is in the immediate grasp is exploring the viability of establishing human statements in
space.
What is in the immediate grasp is to explore the viability of establishing human statements in
space.
What is in the immediate grasping is exploring the viability of establishing human statements in
space.
What is in the immediate grasp is that exploration the viability of establishing human statements
in space.

9/40.
B.
C.
D.

10/40.
A.
B.
C.
D.

As they deplete the levels of oxygen in the waters, all aquatics are endangered by toxic
effluents.
As they depleted the levels of oxygen in the waters, all aquatics are endangered by toxic
effluents.
As they deplete the levels of oxygen in the waters, all toxic effluents endanger aquatic life.
As it depletes the levels of oxygen in the waters, toxic effluents endangers all aquatic life.

11/40.
A.
Extraordinary attention has been devoted to the development of the managerial skills, both
within the company and through participation in programmes sponsored by external bodies.
B.
Extraordinary attention have been devoted to the development of the managerial skills, both
within the company and through participation in programmes sponsored by external bodies.
C.
Extraordinary attention has been devoted to the development of both managerial skills, both
within the company and through participation in programmes sponsored by external bodies.
D.
Extraordinary attention has been devoted to the development of the managerial skills, both
within the company and through participation of programmes sponsored by external bodies.
12/40.
A.
B.
C.
D.

When animals will be faced to threats to their physical survival, they would react by aggression
or by submission.
When animals are faced with threats on their physical survival, they react by aggression or by
submission.
When animals are faced with threats to their physical survival, they react by aggression or by
submission.
When animals are faced with threats to its physical survival, they react by aggression or by
submission.

13/40.
A.
B.
C.
D.

Declining public school quality in addition to funding for public higher education and a failure
of political will make higher education affordable have produced a growing education gap
among the people.
Declining public school quality in addition to funding for public higher education and a failure
of political will to make higher education affordable will be producing a growing education gap
among the people.
Declining public school quality in addition to funding for public higher education and a failure
of political will to make higher education affordable has produced a growing education gap
among the people.
Declining public school quality in addition to funding for public higher education and a failure
of political will make higher education affordable will have producing a growing education gap
among the people.

14/40.
A.
B.
C.
D.

The crisis of drinking water is deepening because water resources are drying up and the
lowering of ground water.
The crisis of drinking water is deepening because of the drying up of water resources and the
lowering of ground water through over-pumping.
The crisis with drinking is deepening because water resources have dried up and the lowering of
ground water through over pumping.
The crisis in drinking water deepens because water resources ried up and the lowering of ground
water through over pumping.

Directions: Read the following passage below and answer the questions that follow on the basis of what is
stated / implied in that passage.
In ancient times the most common form of advertising was by word of mouth; however, commercial messages
and political campaign displays have been found in the ruins of Pompeii. Egyptians used papyrus to
create sales messages and wall posters, while lost-and-found advertising on papyrus was common in
Greece and Rome. wall or rock painting for commercial advertising is another manifestation of an
ancient media advertising form, which is present to this day in many parts of Asia, Africa. and South
America. For instance, the tradition of wall paintings can be traced back to Indian rock-art painting that
goes back to 4000 BC. As printing developed in the 15th and 16th century, advertising expanded to
include handbills. In the 17th century advertisements started to appear in weekly newspapers In
England.
These early print ads were used mainly to promote books which became increasingly affordable thanks to the
printing press and medicines, which were increasingly sought after as disease ravaged Europe. However,
false advertising and so-called 'quack ads became a problem. which ushered in regulation of advertising
content.
As the economy was expanding during the 19th century. the need for advertising grew at the same pace. In the
United States. classified ads became popular, filling pages of newspapers with small print messages

promoting all kinds of goods. The success of this advertising format led to the growth of mall-order
advertising such as the Sears Catalog. at one time referred to as the 'Farmer's Bible'. In 1843 the first
advertising agency was established by Volney Palmer in Philadelphia. At first the agencies were just
brokers for ad space in newspapers, but by the 20th century. advertising agencies started to take over the
responsibility for the content as well.
The 1960s saw advertising transform into a modern. more scientific approach in which creativity was allowed to
shine, producing unexpected messages that made advertisements more tempting to consumers' eyes. The
Volkswagen ad campaign featuring such headlines as 'Think Small" and "Lemon" ushered in the era of
modern advertising by promoting a "position' or 'unique selling proposition' designed to associate each
brand with a specific idea in the reader or viewer's mind.
The tate 1980s and early 1990s saw the introduction or cable television and particularly MN. Pioneering the
concept or the music .video. MT/ ushered In a new type of advertising: the consumer tunes in for the
advertisement. rather than it being a byproduct or afterthought. As cable (and later satellite) television
became increasingly prevalent. "specialty' channels began to emerge. and eventually entire channels.
such as QVC and Home Shopping Network and ShopTv. devoted to advertising merchandise, where
again the consumer tuned in for the ads.
marketing through the Internet opened new frontiers for advertisers and led to the "dot.com' boom of the 1990s.
Entire corporations operated solely on advertising revenue, offering everything from coupons to free
Internet access. At the turn of the list century, search engine Google revolutionized online advertising by
emphasizing contextually relevant. unobtrusive ads intended to help, rather than inundate, users. This
has led to a plethora of similar efforts and an increasing trend of integrated advertising.
Today. there are innovations like "guerrilla" promotions. which Involve unusual approaches such as staged
encounters in public places, giveaways of products such as cars that are covered with brand messages,
and interactive advertising where the viewer can respond to become part of the advertising message.
This reflects an increasing trend of interactive and 'embedded' ads, such as via product placement,
having consumers vote through text messages, and various other innovations utilizing social networking
sites (e.g. myspace).
The same advertising techniques used to promote commercial goods and services can be used to inform. educate
and motivate the public about non-commercial issues, such as AIDS, political ideology, energy
conservation, religious recruitment, and deforestation.
Advertising. in its non-commercial guise, is a powerful educational tool capable of reaching and motivating
large audiences. 'Advertising justifies its existence when used in the public interest - it is much too
powerful a tool to use solely for commercial purposes."
Public service advertising, non-commercial advertising, public interest advertising. cause marketing. and social
marketing are different terms for (or aspects of) the use of sophisticated advertising and marketing
communications techniques (generally associated with commercial enterprise) on behalf of noncommercial, public interest issues and initiatives.
In the United States. the granting of television and radio licenses by the FCC is contingent upon the station
broadcasting a certain amount of public service advertising. To meet these requirements, many broadcast
stations in America air the bulk of their required Public Service Announcements during the late night or
early morning when the sm4est percentage of viewers are watching, leaving more day and prime time
commercial slots available for high-paying advertisers.
15/40. Which of the following is not prevalent in the advertising?
A.
Distribution of handbills to the public.
B.
People talking about the products they used.
C.
Public displays and wall posters.
D.
Wall and rock paintings with messages.
16/40. Why are Public Service Announcements aired mostly during late nights or early mornings in the US?
A.
The viewers dont like watching them.
B.
The prime time slots are reserved by the high paying advertisers.
C.
As it is mandatory to air them to become eligible for licence, the channels air them when their
revenues are least affected.
D.
The TV channels are more concerned about the popularity ratings rather than the revenue.
17/40. It can be inferred from the passage that
A.
Advertising has even a longer way to go.
B.
Mail order advertising is now losing its popularity to the e-advertising.
C.
Advertising hardly educates the public on the essentials of the product advertised.
D.
Advertising is not a modern idea.
18/40. What was the reason for regulation to be introduced in advertising?
A.
There were far too many advertisers.
B.
Disease spread across Europe and people needed to know about the medicine.
C.
Misleading the false advertisements sprang up.

D.

The popularity of advertisements which made people depend on them.

19/40. Which of the following is an example of favourable advertising?


A.
Cable TV advertising.
B.
Integrated advertising through the internet.
C.
Staged encounters in public places.
D.
Advertising through social marketing.
Directions: Fill in the blank with the correct option that fills in grammatically and logically.
20/40. She is such a hospitable person. She ___________ to make us feel welcome.
A.
Goes a long way.
B.
Goes to great heights
C.
Has come a long way.
D.
Makes a long face.
21/40. It all depends now on ______________.
A.
Them coming in time.
B.
Their coming on time.
C.
Them coming with time.
D.
Them coming in time.
22/40. Ill talk to the director _____________.
A.
Within hailing distance.
B.
Waiting in the wings.
C.
When the time is ripe.
D.
Through the thick and thin.
23/40. One way of preventing malaria ______ the anopheies mosquito.
A.
Was for eradicating.
B.
Is to eradicate.
C.
Should be eradication of
D.
Will be for eradicating.
24/40. _____________ a new comer, I was unable to get a suitable house for rent.
A.
Due to me being.
B.
Owing to my being.
C.
Since being.
D.
Being.
25/40. The man was advised by the doctor to ________ from alchohol.
A.
abstain.
B.
Refrain.
C.
Keep out of.
D.
Not to indulge.
26/40. As I was walking home late in the night, a car _____ and the driver offered me a lift.
A.
Had stopped.
B.
Having stopped.
C.
Was stopping me.
D.
Stopped.
27/40. __________ to look after his own interests in this matter.
A.
He advised each of them.
B.
He was advising to each of them.
C.
He told all of them.
D.
He said to them all.

Directions: in each of the following sentences, some part of the sentence or the entire sentence Is underlined.
Beneath each sentence you will find four ways of phrasing the underlined part. This is a test of
correctness and effectiveness of expression. In choosing answers, follow the requirements of standard
written English, that is, pay attention to grammar, choice of words, and sentence construction. Choose
the answer that expresses most effectively what is presented in the original sentence; this answer should
be clear and exact, without awkwardness, ambiguity or redundancy.
28/40. Floriculture is a discipline of horticulture concerned with the cultivation of flowering and ornamental
plants, which are cultivated all around the world both indoors and outdoors.
A.
That are cultivating all over the world both indoors and outdoors.
B.
Which are cultivated all the world both indoors and outdoors.
C.
That are cultivated both indoors and outdoors.
D.
That is cultivated all across the world, both indoors and outdoors.
29/40. Hedging is a strategy designed to minimize exposure to and unwanted business risk, by still allowing the
business to profit from an investment activity.
A.
Over an unwanted business risk, though still allowing the business to profit against an.
B.
by an unwanted business risk, when still allowing the business to profit from an.
C.
To a unwanted business risk, through still allowing the business to profit from an.
D.
To an unwanted business risk, through still allowing the business to profit from an.
30/40. The refrigerator is allowing the modern family to purchase, to store , freeze ,prepare and preserve food
products not generally found available in the natural locale.
A.
Has allowed the modern family to purchase, to store, freeze, prepare and to preserve.
B.
Has allowed the modern family to purchase, store, freeze, prepare and preserve.
C.
will allow the modern family to purchase, store, freeze, prepare and to preserve.
D.
Has allowed the modern family to purchase, store, freeze, prepare and preserve.
31/40. Myths are generally narratives about divine or heroic beings, arranged in a coherent system, passed down
traditionally, and being linked to the spiritual or religious life of a community, endorsed by rulers or
priests.
A.
And linking the spiritual or religious life of a community, endorsed with rulers or priests.
B.
or linked the spiritual or religious life of a community, endorsed by rulers or priests.
C.
And linked the spiritual or religious life of a community, endorsed for rulers or priests.
D.
And linked the spiritual or religious life of a community, endorsed by rulers or priests.
32/40. Nargisis an indian actress is best known for her role as Radha in the Oscar nominated film
Mother India.
A.
Was an indian actress, best known for her role as.
B.
Is the indian actress, best known for her role as.
C.
Was one of the indian actress, best known for her role as.
D.
Was an indian actress, most known for her role as.
33/40.Depending on the herbs special properties, it can be used to soothe or tone muscles, to ease a tired mind,
or to just make people feel beautiful.
A.
Depending on a herbs special properties, they can be used to
B.
Depending on a herbs special properties, it can be used to
C.
Dependent upon a herbs special properties, it can be used to
D.
Depending from a herbs special properties, it can be used to
34/40.Bankers acceptances are considered very safe assets as it allows traders to substitute the banks credit
standing for their own.
A.
Bankers acceptances are considered very safe assets as it allows traders.
B.
Bankers acceptances are considered very safe assets as it allow traders.
C.
Bankers acceptances are considered very safe assets as these allow traders.
D.
Banker acceptances are considered very safe assets as it allows traders.
35/40. Tom and Jerry are an animated cat(Tom) and mouse(Jerry) team who has formed the basis of a massively
successful series of theatrical, short animated films created, written and directed by animators, William
Hanna and Joseph Barbara.
A.
Which formed the basis of a massively successful series.
B.
Whos formed the basis of a massively successful series.
C.
Who have formed the basis of a massively successful series.
D.
Who form the basis of a massively successful series.
Directions: read each of the following passage below and answer the questions that follow on the basis
of what is started / implied in that passage.

There was a time when being a teacher at Lenawee Intermediate School District (MI) meant getting in linea
long lineif you wished to get hold of audiovisual equipment for your class. Teachers had to reserve the
equipment weeks in advance. Next, once the time came to use it, representatives from the district's A/V
department had to quite literally schlep it all to the dassroom themselves. When instructors were finished
with it, they called the A/V guys to come on back and tear the equipment down. Depending on the setup
a particular teacher required, the process could take hours. What's more, it was tedious workthe worst
kind of obstacle in a high-pressure environment.
Last year, the arrival of media carts changed the whole routine. Clustering certain equipment and placing it as a
unit on a refrigerator sized wheeled cart revolutionized Lenawee's A/V delivery system. Yes, teachers
still have to reserve the media, and yes, A/V technicians still have to be the ones to transport it. But with
laptops, projectors, document cameras, televisions, and VCRs affixed to movable carts, technicians
simply wheel the equipment into a classroom, and everything a teacher needs is right there.
Lucas Wilson, one of the district's technology support specialists, says the new system makes the entire A/V
process more palatable and saves him and his colleagues tons of time. This Isn't exactly emerging
technology or cutting edge, but it brings a lot of different pieces of equipment together and provides
easy access to it, he says, noting that Lenawee now owns three carts in each of its three schools. 'Carts in
our district have been huge." 4 They've been huge in many other districts, as well. At a time when media
technologies are proliferating like gremlins, an increasing number of elementary and secondary school
districts are turning to these wheeled wonders to solve some of their logistical woes. Media cart vendors
such as Bretford, Jar Systems, EarthWalk Communications, Hewlett- Packard, and Span all say cart
sales have increased in recent years. Carts don't merely help organize heavy and hard-to-transport
equipment; they also provide a tidy solution for storing equipment, not to mention a manageable way to
distribute limited resources around a particular school or wing. The best part , of course is that the carts
themselves are relatively inexpensive and frequently come at a discount for those willing to buy enough
media to qualify.
36/40. According to the passage, the teachers job at Lenawee intermediate school was tedious because?
A.
They had to reserve the audio-visual aid much in advance.
B.
They had to fit the equipment in the classroom themselves.
C.
They worked in a high pressure environment.
D.
Both 1st and 2nd options mentioned above.
37/40. The phrase proliferating like gremlins in the passage means?
A.
Multiplying as in cell division.
B.
Mushrooming indiscriminately.
C.
Growing rapidly.
D.
Dividing fast.
38/40. The best title for the passage is?
A.
Wheeled wonders
B.
Media cart- the facilitating technology
C.
Booming technology
D.
Zooming wheels.
39/40. The word schlep in the paragraph 1 is closest in meaning to?
A.
Carry.
B.
Slave.
C.
Wheel.
D.
Drag with difficulty.
40/40. Which of the following is untrue about the new cart technology?
A.
They help solve logistic woes.
B.
They help distribute limited resources.
C.
They simplify storing.
D.
They are popular only in books

Вам также может понравиться